You are on page 1of 231

Joints

University of Pennsylvania Department of Orthopaedic Surgery


2012-04 (Joints)

• Figures 4a and 4b are the radiographs of a man with an 8-


month history of pain and deformity since knee
replacement. He has not had any systemic symptoms and
his erythrocyte sedimentation rate and C-reactive protein
levels are within defined limits. The most appropriate next
step should consist of?

• 1. revision total knee.


• 2. formal physical therapy.
• 3. bracing with a hinged knee brace.
• 4. aspiration and culture of the synovial fluid.
• 5. reconstruction of the lateral collateral ligament.
2012-04 (Joints)
2012-04 (Joints)

• Figures 4a and 4b are the radiographs of a man with an 8-


month history of pain and deformity since knee
replacement. He has not had any systemic symptoms and
his erythrocyte sedimentation rate and C-reactive protein
levels are within defined limits. The most appropriate next
step should consist of?

• 1. revision total knee.


• 2. formal physical therapy.
• 3. bracing with a hinged knee brace.
• 4. aspiration and culture of the synovial fluid.
• 5. reconstruction of the lateral collateral ligament.
• This total knee is clearly subluxed and you
can see the outline of the poly component
has translated out anterior. Highly
unlikely to be infected given normal labs,
so no need for aspiration. No role for
nonoperative management. The only fix is
a revision.
2012-16 (Joints)

• At a mean follow-up of 30 years, what are the


causes of failure requiring revision of the
Charnley lowfriction arthroplasty (most
frequent to least frequent)?
– 1. Deep infection, acetabular, both component, femoral
– 2. Acetabular, deep infection, both component, femoral
– 3. Acetabular, both component, deep infection, femoral
– 4. Both component, deep infection, acetabular, femoral
– 5. Femoral, acetabular, deep infection, both component
Question 16

• At a mean follow-up of 30 years, what are the


causes of failure requiring revision of the
Charnley lowfriction arthroplasty (most
frequent to least frequent)?
– 1. Deep infection, acetabular, both component, femoral
– 2. Acetabular, deep infection, both component, femoral
– 3. Acetabular, both component, deep infection, femoral
– 4. Both component, deep infection, acetabular, femoral
– 5. Femoral, acetabular, deep infection, both component
Question 16

• In 2009 Wroblewski et al published the 30


year follow up data for 110 patients that had
“low-friction” total hip arthoplasty.
– 13/110 required revision (11.8%) and the
breakdown is as follows:
–5 Acetabular component
–4 Loosening both components
–2 Deep infection
–1 Loose femoral component
–1 Fractured femoral component
2012-39 (Joints)
Figures 39a and 39b are the radiographs of a 36-year-old man with sickle cell
anemia who has an 8-month history of right groin and knee pain that started after
tripping and stepping hard onto his right leg. The radiographic changes are most
likely caused by?
 
1. trauma.
2. infection.
3. hemarthroses.
4. vascular occlusion.
5. collapse of an enchondroma.
4. Vascular occlusion. X-rays show flattening of the femoral head. Sickle
cell disease and AVN of the hip is caused by vascular occlusion. Decreased
blood flow to the femoral head leads to avascular necrosis and eventual
collapse of the femoral head. Hemarthroses are seen in bleeding disorders
such as hemophilia, not sickle cell. The patient does not have any clinical
signs/symptoms of infection in the hip. Enchondromas appear in the bone as
stippled calcification. The trauma the patient sustained may have caused
collapse of a non-collapsed lesion in the femoral head but the lesion is due to
vascular occlusion and NOT trauma. When patient’s complain of groin pain it is
usually coming from the hip. Knee pain can be referred pain from the hip.
2012-50 (Joints)
• A 71-year-old farmer had right total hip arthroplasty 2 years ago. He did
well for 6 months and then developed pain in his right hip. He has pain when
rising from a seated position for the first few steps and after a full day of
activities. He denies fevers or chills and the incision healed well. Laboratory
studies show an erthyrocyte sedimentation rate of 8 mm/h (reference range,
0-20 mm/h), C-reactive protein of 3.0 mg/L (reference range, 0-3.0 mg/L),
and white blood cell count of 7.7/mm3 (reference range, 3.5 -10.5/mm3).
No fluid was obtained during aspiration. Figures 50a and 50b show the
immediate postoperative radiographs and Figures 50c and 50d reveal the
most recent radiographs. What is the best definitive management option?

• 1. Revision of the femoral and acetabular component


• 2. Revision of the femoral component to a long cemented revision stem
• 3. Revision of the femoral component to an uncemented diaphyseal
engaging stem
• 4. Irrigation and debridement with the femoral head and liner exchange
• 5. Resection with placement of the antibiotic spacer, 2-stage reimplantation
Question 50: Images
Question 50: Answer

• The preferred response is choice 3- revision


of the femoral component to an uncemented
diaphyseal engaging stem.
Question 50: Explanation

• The clinical scenario and imaging in this question are classic for
loosening of the femoral stem. Start-up pain and absence of any
laboratory markers of infection, with subsidence on the
radiographs, as well as a reactive pedestal, all point to loosening.
The lateral image reveals the cup to be appropriately anteverted.
This patient should undergo a revision of his stem. Proximal
engaging stems should be revised to stems that will obtain fit in
the diaphysis. Cement is rarely used in this setting, as a scratch fit
in the diaphysis will provide better long term stability.
• RECOMMENDED READINGS:
• Sporer SM, Paprosky WG. Revision total hip arthroplasty: the limits
of fully coated stems. Clin Orthop Relat Res. 2003 Dec;(417):203-
9. PubMed PMID: 14646718.
• Moreland JR, Moreno MA. Cementless femoral revision arthroplasty
of the hip: minimum 5 years followup. Clin Orthop Relat Res. 2001
Dec;(393):194-201. PubMed PMID: 11764349.
2012-67 (Joints)
A 61-year-old woman has pain in her right hip after a fall. She had a right total
hip arthroplasty 15 years ago for osteoarthritis secondary to dysplasia. Figures
67a and 67b are radiographs taken after the fall. What is the best treatment
option?

1. Hemiarthroplasty
2. Twelve weeks of nonweight-bearing activity
3. A large hemispherical cup
4. A posterior plate and porous metal cup
5. Medial cancellous grafting with a hemispherical cup
Question 67

University of Pennsylvania Department of Orthopaedic Surgery


Question 67

A 61-year-old woman has pain in her right hip after a fall. She had a right total
hip arthroplasty 15 years ago for osteoarthritis secondary to dysplasia. Figures
67a and 67b are radiographs taken after the fall. What is the best treatment
option?

1. Hemiarthroplasty
2. Twelve weeks of nonweight-bearing activity
3. A large hemispherical cup
4. A posterior plate and porous metal cup
5. Medial cancellous grafting with a hemispherical cup
 
RECOMMENDED READINGS:
DeBoer DK, Christie MJ, Brinson MF, Morrison JC. Revision total hip
arthroplasty for pelvic discontinuity. J Bone Joint Surg Am. 2007 Apr;89(4):835-
40. PubMed PMID: 17403808.
Paprosky WG, O’Rourke M, Sporer SM. The treatment of acetabular bone
defects with an associated pelvic discontinuity. Clin Orthop Relat Res. 2005
Dec;441:216-20. PubMed PMID: 16331006.
 

University of Pennsylvania Department of Orthopaedic Surgery


Question 67

A 61-year-old woman has pain in her right hip after a fall. She had a right total
hip arthroplasty 15 years ago for osteoarthritis secondary to dysplasia. Figures
67a and 67b are radiographs taken after the fall. What is the best treatment
option?

1. Hemiarthroplasty
2. Twelve weeks of nonweight-bearing activity
3. A large hemispherical cup
4. A posterior plate and porous metal cup
5. Medial cancellous grafting with a hemispherical cup

EXPLANATION:
The radiograph in this question shows a fracture line at the superomedial aspect of the cup consistent
with pelvic discontinuity. This is the most severe of defects when it comes to acetabular defects in
revision THA. Paprosky et al. reviewed patients who had an acetabular revision using a trabecular
metal acetabular component for a pelvic discontinuity and compared these patients with a cohort of
patients who had a previous reconstruction for a pelvic discontinuity using an acetabular cage. They
found a decreased incidence of pain or need for walking aids in patients who had revision with a
trabecular metal acetabular component.  DeBoer et al. describe the results of 28 patients with pelvic
discontinuity treated with a custom-made porous-coated triflange acetabular prosthesis.
Hemiarthroplasty is useless as the patient has an acetabular defect. NWB is stupid when you consider
the severity of the defect. And a large cup even with grafting isn’t addressing the discontinuity.

University of Pennsylvania Department of Orthopaedic Surgery


 
2012-78 (Joints)

• During implantation of a tapered, proximally


coated femoral stem, the stem implant
subsided during final impaction and a 1-cm
longitudinal split was discovered in the calcar.
What is the best treatment option?
– 1 – Stem removal, cabling, and reinsertion
– 2 – Stem removal and use of a cemented stem
– 3 – Conversion to a long, revision full-coat stem
– 4 – Cable placement around the stem in the
subsided position
– 5 – Leave the stem in place and order nonweight-
bearing activity for 6 weeks
Question 78

• During implantation of a tapered, proximally


coated femoral stem, the stem implant
subsided during final impaction and a 1-cm
longitudinal split was discovered in the calcar.
What is the best treatment option?
– 1 – Stem removal, cabling, and reinsertion
– 2 – Stem removal and use of a cemented stem
– 3 – Conversion to a long, revision full-coat stem
– 4 – Cable placement around the stem in the
subsided position
– 5 – Leave the stem in place and order nonweight-
bearing activity for 6 weeks
Question 78

• Intraoperative fractures of the proximal


femur generally occur during rasping
(boraching) or implant insertion. Berend et al.
reviewed a series of 58 total hip
arthroplasties who sustained an
intraoperative calcar fracture. All were
treated with cable wiring of the calcar and
stem insertion. The authors report no femoral
component subsidence or failure otherwise at
16 year follow-up.
2012-92 (Joints)

The implant shown in Figures 92a and 92b was


one of the earliest attempts to manufacture a total knee
arthroplasty. What most likely led to its early mechanical
failure?

• 1. Lack of high flexion


• 2. Metal-on-metal wear debris
• 3. Loosening attributable to overconstraint
• 4. Wear of polyethylene irradiated in air
• 5. Absence of a patella-femoral resurfacing

University of Pennsylvania Department of Orthopaedic Surgery


University of Pennsylvania Department of Orthopaedic Surgery
Joints

Question 92
The implant shown in Figures 92a and 92b was
one of the earliest attempts to manufacture a total knee
arthroplasty. What most likely led to its early mechanical
failure?

• 1. Lack of high flexion


• 2. Metal-on-metal wear debris
• 3. Loosening attributable to overconstraint
• 4. Wear of polyethylene irradiated in air
• 5. Absence of a patella-femoral resurfacing

University of Pennsylvania Department of Orthopaedic Surgery


Explanation
• Deciding the amount of constraint to use in a particular TKA is an
important, yet challenging, element of preoperative planning. Using
an implant with insufficient constraint risks failure from instability,
whereas using a device that has more constraint than is necessary
can predispose the patient to aseptic loosening and bone loss. Early
attempts at knee arthroplasty provided more constraint than
necessary and led to early failure secondary to aspetic loosening.

RECOMMENDED READINGS:
Lombardi AV Jr, Berend KR. Posterior cruciate ligament-retaining,
posterior stabilized, and varus/valgus posterior stabilized constrained
articulations in total knee arthroplasty. Instr Course Lect. 2006;55:419-
Review. PubMed PMID: 16958477.

Morgan H, Battista V, Leopold SS. Constraint in primary total knee


arthroplasty. J Am Acad Orthop Surg. 2005 Dec;13(8):515-24. Review.
PubMed PMID: 16330513.

University of Pennsylvania Department of Orthopaedic Surgery


2012-105 (Joints)

• During revision total knee arthroplasty, difficulty was


encountered while attempting to gain adequate
exposure. The medial parapatellar arthrotomy was
extended proximally and a lateral release was
performed. The tibia was externally rotated with a
medial release. Exposure of the knee was still not
adequate to safely perform the revision. What is the
best option to enhance exposure?
• 1. Quadriceps snip
• 2. Patellar eversion
• 3. V-Y turndown of the extensor mechanism
4. Tibial tubercle osteotomy with internal fixation
• 5. Patellar turndown (modified Coonse-Adams
approach)

University of Pennsylvania Department of Orthopaedic Surgery


Question 105

• During revision total knee arthroplasty, difficulty was


encountered while attempting to gain adequate
exposure. The medial parapatellar arthrotomy was
extended proximally and a lateral release was
performed. The tibia was externally rotated with a
medial release. Exposure of the knee was still not
adequate to safely perform the revision. What is the
best option to enhance exposure?
• 1. Quadriceps snip
• 2. Patellar eversion
• 3. V-Y turndown of the extensor mechanism
4. Tibial tubercle osteotomy with internal fixation
• 5. Patellar turndown (modified Coonse-Adams
approach)

University of Pennsylvania Department of Orthopaedic Surgery


Question 105

• Adequate exposure is a prerequisite for revision


TKA
• Flexion of 110 is desirable to allow safe delivery
of the components and flexion <90 under GA
indicates that a simple medial parapatellar
approach may be inadequate
• Meek et al demonstrated that there is no
statistical difference in WOMAC function, pain,
stiffness, and satisfaction scores when the rectus
snip is compared to medial parapatellar approach

University of Pennsylvania Department of Orthopaedic Surgery


2012-128 (Joints)
Q128 Images
Q128 Answers
Q128 Explanations

• Anticipated change in leg length is


determined by evaluating the vertical
distance between the center of the acetabular
component and the center of the femoral
head.
• Offset is measured from center of the head to
the tip of the ipsilateral greater trochanter.
2012-137 (Joints)

What is considered to be an absolute indication for patellar


resurfacing during total knee arthroplasty?

1. Patella thickness is 16 mm
2. Patella diameter is 22 mm
3. A patient’s BMI is >40
4. A patient has rheumatoid arthritis
5. An eburnated trochlea is present

University of Pennsylvania Department of Orthopaedic Surgery


Question 137

What is considered to be an absolute indication for patellar


resurfacing during total knee arthroplasty?

1. Patella thickness is 16 mm
2. Patella diameter is 22 mm
3. A patient’s BMI is >40
4. A patient has rheumatoid arthritis
5. An eburnated trochlea is present

University of Pennsylvania Department of Orthopaedic Surgery


Question 137

4. A patient has rheumatoid arthritis

The role patellar resurfacing in total knee arthoplasty is controversial.

Indications for leaving the patella unresurfaced are a primary diagnosis of osteoarthritis,
satisfactory patellar cartilage with no eburnated bone, congruent patellofemoral
tracking, normal anatomical patellar shape, and no evidence of inflammatory or
crystalline arthropathy.

Patellar resurfacing is indicated in patients with rheumatoid arthritis (or other


inflammatory arthropathies), largely owing to the fact that the rheumatoid patella is
often eburnated and mal-shapen, with the potential for mal-tracking and pain if
not surgically addressed.

Obesity, severe OA (Gr 3/4), and elderly patients, are relative indications.

RECOMMENDED READINGS:
Burnett RS, Bourne RB. Indications for patellar resurfacing in total knee arthroplasty. Instr Course Lect.
2004;53:167-86. PMID: 15116611.
Holt GE, Dennis DA. The role of patellar resurfacing in total knee arthroplasty. Clin Orthop Relat Res.
2003 Nov;(416):76-83. PubMed PMID: 14646743.

University of Pennsylvania Department of Orthopaedic Surgery


2012-149 (Joints)

Figures 149a and 149b are the radiographs of a 55-year-


old man with left hip pain, popping, and grinding since
having a metal-on-metal left total hip arthroplasty 18
months ago. He has had 3 dislocations during the last 8
months. What is the best treatment option?

1. Both component revision


2. Acetabular component revision and femoral head
exchange
3. Exchange of the liner to a face-changing liner and a
larger femoral head
4. Exchange of the head and liner to a polyethylene liner
and ceramic femoral head
5. Resection arthroplasty with an antibiotic spacer and
2-stage reimplantation
Question 149
Question 149

Figures 149a and 149b are the radiographs of a 55-year-


old man with left hip pain, popping, and grinding since
having a metal-on-metal left total hip arthroplasty 18
months ago. He has had 3 dislocations during the last 8
months. What is the best treatment option?

1. Both component revision


2. Acetabular component revision and femoral head
exchange
3. Exchange of the liner to a face-changing liner and a
larger femoral head
4. Exchange of the head and liner to a polyethylene liner
and ceramic femoral head
5. Resection arthroplasty with an antibiotic spacer and
2-stage reimplantation
This patient has a malpositioned acetabular component with anteversion
<0 degrees which in a study by von Knoch et al (2002) was found to be
associated with increased risk of dislocation.

Wrong Answers
1. His femoral stem does not show signs of loosening and therefore both
component revision is not indicated
3. and 4. Study by Sultan et al (2002) from Penn demonstrated that a 15
deg elevated-rim acetabular liner in the posterior quadrant increased hip
stability by an additional 8.1 to 8.9 degrees of internal rotation
depending on head size. 32mm head may also contribute to hip stability.
This patient has a metal on metal implant with a malpostioned cup
requiring revision.
5. There is no indication from the question stem that this patient has
infection requiring 2 stage revision.
Question 149

RECOMMENDED READINGS:

von Knoch M, Berry DJ, Harmsen WS, Morrey BF. Late


dislocation after total hip arthroplasty. J Bone Joint
Surg Am. 2002 Nov;84-A(11):1949-53. PubMed
PMID: 12429754.

Sultan PG, Tan V, Lai M, Garino JP. Independent


contribution of elevated-rim acetabular liner and
femoral head size to the stability of total hip implants.
J Arthroplasty. 2002 Apr;17(3):289-92. PubMed
PMID: 11938503.
2012-159 (Joints)

• Who was the first surgeon to develop and use a


stemmed implant that replaced the femoral head
with metal?

1. Otto E. Aufranc
2. John Charnley
3. Austin T. Moore
4. Maurice Muller
5. Marius Smith-Peterson
Hip and Knee Reconstruction

• Who was the first surgeon to develop and use a


stemmed implant that replaced the femoral head
with metal?

1. Otto E. Aufranc
2. John Charnley
3. Austin T. Moore
4. Maurice Muller
5. Marius Smith-Peterson
History of Hip Replacement
• 1923: Marius Nygaard Smith-Peterson
(acetabular mold made of vitallium after
unsuccessful implants with Bakelite and
glass)
• 1939: Austin Moore and Frederick
Thomson independently developed
prostheses that replaced the femoral head
• 1950s: Otto Aufranc (Smith-Peterson
student) improved the vitallium mold design
• 1962: John Charnley (total hip
arthroplasty, plastic cup, metal head)
• 1960s: Maurice Muller (curved stem to
eliminate need for troch osteotomy, also co-
founded AO)

Austin-Moore Prosthesis
2012-170 (Joints)

In the absence of infection as seen in Figures 170a through 170d,


which femoral bone loss classification and treatment option best
describes this scenario?
 
1. Paprosky IIIA - distal modular tapered stem
2. Paprosky IIIB - proximal modular stem
3. Paprosky IIIB - 10-inch extensively coated stem
4- Paprosky IV - 10-inch extensively coated stem
5- Paprosky IV - femoral replacing endoprosthesis
170
170

In the absence of infection as seen in Figures 170a through 170d,


which femoral bone loss classification and treatment option best
describes this scenario?
 
1. Paprosky IIIA - distal modular tapered stem
2. Paprosky IIIB - proximal modular stem
3. Paprosky IIIB - 10-inch extensively coated stem
4- Paprosky IV - 10-inch extensively coated stem
5- Paprosky IV - femoral replacing endoprosthesis
170 - Explanation

• Here is the Paprosky classification:


– Type I: Minimal loss of metaphyseal cancellous bone. Intact diaphysis. Consider cemented vs. cementless
fixation.
– Type II: Extensive loss of metaphyseal cancellous bone. Intact diaphysis. Loss of cancellous bone makes
cemented fixation more suspect, consider uncemented fixation (e.g. fully porous coated stem)
– Type III-A: The metaphysis is not supportive. There remains greater than 4 cm of bone in the diaphysis to allow
for a scratch fit. Consider uncemented fixation with a fully porous-coated stem vs. a modular tapered stem.
– Type III-B: The metaphysis is not supportive. There remains less than 4 cm of bone in the diaphysis to allow for a
scratch fit.  Due to short segment of cylindrical bone to support a fully-porous coated stem, the failure rate is high
with such a device. Consider a modular tapered stem.
– Type IV: Wide open canal without any appreciable isthmus to support an uncemented stem. Consider impaction
grafting if the proximal tube is intact +/- an intact calcar.  Other alternatives would include an APC or a modular
tumor megaprosthesis.

• The images show that there is extensive bone loss and wide
canals all the way down to the distal shaft. An uncemented stem
cannot be supported. Therefore, a tumor prosthesis would need
to be utilized. In some cases of type IV bone loss, and modular
tapered stem can be used.
2012-177 (Joints)

• Which cup position in a metal-on-metal hip arthroplasty is most commonly


associated with elevated serum metal ion levels?

• 1. Anteversion >20 degrees


• 2. Anteversion <20 degrees
• 3. Cup abduction <35 degrees
• 4. Cup abduction >55 degrees
• 5. Cup abduction of 45 degrees
• Question 177
• Which cup position in a metal-on-metal hip arthroplasty is most commonly
associated with elevated serum metal ion levels?

• 1. Anteversion >20 degrees


• 2. Anteversion <20 degrees
• 3. Cup abduction <35 degrees
• 4. Cup abduction >55 degrees
• 5. Cup abduction of 45 degrees
• De Hann, et al, looked at 214 patients with metal on metal hip replacements and
categorized them based on abduction angle of the acetabular cup (greater than 55
degrees vs less than 55 degrees). They found significantly greater metal ions in patients
with abduction angles greater than or equal to 55 degrees. They also found significantly
higher ion levels in patients with arcs of cover of less than 10mm. They attributed higher
level of ions with abduction angles greater than 55 degrees and arcs of cover less than 10
to increased risk of edge-loading leading to marked localized wear. They also found that
women were at higher risk of elevated metal ions with steep (>55 degrees) implants
(20% of women vs 2% of men with >55 degrees). However, they did not find a
correlation between the patient’s activity level and their concentration of metal ions.
• Desy, et al, also showed a positive correlation in increasing metal ions with an increasing
inclination angle. They also found that smaller femoral heads were associated with larger
metal ion levels. They did not find a correlation between severity of pre op OA,
acetabular version, femoral stem-shaft and valgus angle, and anterior orientation of
femoral component and the circulating metal ion levels.

• Desy NM, Bergeron SG, Petit A, Huk OL, Antoniou J. Surgical variables influence metal ion levels after hip resurfacing. Clin Orthop Relat Res. 2011
Jun;469(6):1635-41. PubMed PMID: 20972653.
• De Haan R, Pattyn C, Gill HS, Murray DW, Campbell PA, De Smet K. Correlation between inclination of the acetabular component and metal ion levels in
metal-on-metal hip resurfacing replacement. J Bone Joint Surg Br. 2008 Oct;90(10):1291-7. Erratum in: J Bone Joint Surg Br. 2009 May;91(5):700.
PubMed PMID: 18827237.
• Langton DJ, Sprowson AP, Joyce TJ, Reed M, Carluke I, Partington P, Nargol AV. Blood metal ion concentrations after hip resurfacing arthroplasty: a
comparative study of articular surface replacement and Birmingham Hip Resurfacing arthroplasties. J Bone Joint Surg Br. 2009 Oct;91(10):1287-95.
PubMed PMID: 19794161.
2012-187 (Joints)

During total knee arthroplasty, after placement of


the trials, the patella subluxates laterally out of the
trochlea. The axial alignment is appropriate. What
is the next best step to improve patella tracking?

1. Tibial tubercle transfer


2. Additional resection of the patella
3. Downsizing the femoral component
4. External rotation of the tibial component
5. Internal rotation of the femoral component

University of Pennsylvania Department of Orthopaedic Surgery


Question 187 – Preferred Response

During total knee arthroplasty, after placement of


the trials, the patella subluxates laterally out of the
trochlea. The axial alignment is appropriate. What
is the next best step to improve patella tracking?

1. Tibial tubercle transfer


2. Additional resection of the patella
3. Downsizing the femoral component
4. External rotation of the tibial component
5. Internal rotation of the femoral component

University of Pennsylvania Department of Orthopaedic Surgery


Question 187 - Explanation

Patellar tracking is largely dependent upon the Q-angle, which


helps describe the resultant force on the patella created by the
pull of the quadriceps. Normal Q-angle for males is 14-degrees
and for females is 17-degrees (Agliettis et. al, CORR 1983). In
TKA, the prosthetic patella is less restrained than in a native knee
and is thus more dependent upon Q-angle for optimal tracking.
Causes of patellar maltracking in TKA are varus cuts (leading to
valgus knee alignment) IR and/or medialization of either the
femoral or tibial components. Bengs and Scott demonstrated
that while increasing patellar thickness limits passive knee flexion
(3-degrees per 2mm thickness) it had no affect on patellar
tracking.

McPherson EJ. Patellar tracking in primary total knee arthroplasty. Instr Course Lect.
2006;55:439-48. Review. PubMed PMID: 16958479.

Bengs BC, Scott RD. The effect of patellar thickness on intraoperative knee flexion and
patellar tracking in total knee arthroplasty. J Arthroplasty. 2006 Aug;21(5):650-5.
PubMed PMID: 16877149.

University of Pennsylvania Department of Orthopaedic Surgery


2012-196 (Joints)

When using highly cross-linked ultra-high


molecular-weight polyethylene acetabular liners in
primary total hip arthroplasty, what is the optimal
head size in relation to wear rates?
 
1. 22 mm
2. 28 mm
3. 32 mm
4. 40 mm
5. Head size does not affect wear rates.
Question 196
When using highly cross-linked ultra-high
molecular-weight polyethylene acetabular liners in
primary total hip arthroplasty, what is the optimal
head size in relation to wear rates?

Excluded from final grading


 
1. 22 mm
2. 28 mm
3. 32 mm
4. 40 mm
5. Head size does not affect wear rates.
• In-vivo studies have shown very low wear rates using highly cross-linked
UHMWPE bearing surfaces with 28 and 32 mm cobalt-chrome femoral
heads. Hip simulator studies with femoral heads of different diameters
(range, 22–40 mm) have also demonstrated very low wear rates occur
even with large heads, and wear with cobalt-chrome femoral heads paired
against electron beam highly cross linked UHMWPE was independent of the
head size.
• The decrease in wear appears to be independent of head size with a highly
cross linked polyethylene studied in vitro, whereas larger heads produced
more wear when articulated with conventional UHMWPE
• Dislocation is often secondary to impingement. Impingement can occur
between osseous structures or between implant components, which can
lead to levering of the femoral head out of the socket. The type of
impingement depends on the size of the femoral head. With 22 mm
femoral heads, impingement between components occurs, most often
between the femoral neck and the acetabular component. With femoral
heads larger than 32 mm in diameter, bony impingement commonly
occurs between the proximal femur and pelvis.
• The jump distance (the distance the head must travel to disengage from
the socket) is higher with a larger femoral head, which is more stable
against dislocation.
2012-205 (Joints)

# 205: A 62-year-old man has moderate knee osteoarthritis that


interferes with golfing. He has a history of type II diabetes,
obesity (body mass index 37), and cardiac stenting. What do the
2011 American Academy of Orthopaedic Surgeons Guidelines
recommend for the treatment of his osteoarthritis?
 
1. Weight loss
2. Arthroscopy
3. Chondroitin sulfate
4. Lateral heel wedges
5. Hyaluronic acid injections
OITE 2012
#205
# 205: A 62-year-old man has moderate knee osteoarthritis that
interferes with golfing. He has a history of type II diabetes,
obesity (body mass index 37), and cardiac stenting. What do the
2011 American Academy of Orthopaedic Surgeons Guidelines
recommend for the treatment of his osteoarthritis?
 

1. Weight loss
2. Arthroscopy
3. Chondroitin sulfate
4. Lateral heel wedges
5. Hyaluronic acid injections
OITE 2012
#205
AAOS Knee OA Reccomendations:
1. Pts w/ symptomatic knee OA participate in low impact aerobic exercises, strengthening, self management: Strong

2. weight loss for patients with symptomatic OA of the knee and a


BMI ≥ 25: Moderate
3. A: Can NOT recommend acupuncture of Knee: Strong
3B: unable to recommend for or against the use of physical agents (including electrotherapeutic modalities): Inconclusive
3C: unable to recommend for or against manual therapy: inconclusive

4. Unable to recommend for or against use of valgus directing force brace (medial compartment unloader): Inconclusive
5. Can not suggest that lateral wedge insoles be used for patients with symptomatic medial compartment OA of the knee: Moderate
6. cannot recommend using glucosamine and chondroitin for patients with symptomatic OA of the knee: Strong
7. A. Recommend NSAIDS, and Tramadol: Strong
7B:unable to recommend for or against the use of acetaminophen, opioids, or pain patches for patients w/symptomatic
osteoarthritis of the knee: Inconclusive
8. Unable to recommend for or against Intra-articular steroids: Inconclusive
9. Can Not recommend use of Hyaluronic Acid intra articular injections: Strong
10. unable to recommend for or against growth factor injections and/or platelet rich plasma: Inconclusive
11. cannot suggestthat the practitioner use needle lavage for patients w/ Sympomatic OA: moderate
12. cannot recommend performing arthroscopy with lavage and/or debridement in patients with a primary diagnosis of Knee OA
13. unable to recommend for or against arthroscopic partial meniscectomy in patients with osteoarthritis of the knee with a torn
meniscus: Inconclusive
14: practitioner might perform a valgus producing proximal tibial osteotomy in patients with symptomatic medial compartment
osteoarthritis of the knee.: Limited
15: n the absence of reliable evidence, it is the opinion of the work group not to use the freefloating (unfixed) interpositional device in
patients with symptomatic medial compartment osteoarthritis of the knee: Consensus
2012-225 (Joints)

The use of a continuous passive motion device after


total knee replacement results in?
 
1. increased blood loss.
2. improved 6-month quad strength.
3. improved 6-month range of motion.
4. improved early active knee flexion.
5. decreased incidence of deep venous thrombosis.

University of Pennsylvania Department of Orthopaedic Surgery


Question 225

The use of a continuous passive motion device after


total knee replacement results in?
 
1. increased blood loss.
2. improved 6-month quad strength.
3. improved 6-month range of motion.
4. improved early active knee flexion.
5. decreased incidence of deep venous thrombosis.

University of Pennsylvania Department of Orthopaedic Surgery


Lotke demonstrated no significant difference in TKA blood loss
between patients treated with CPM versus patients treated
without CPM. In his meta-analysis, Bourne found evidence
suggesting CPM can shorten the length of hospital stay and
improve knee flexion at early time points, but did not find any
evidence of improved clinical outcome scores. The correct answer
is therefore (4) improved early active knee flexion.

RECOMMENDED READINGS:
Lotke PA, Faralli VJ, Orenstein EM, Ecker ML. Blood loss after total
knee replacement. Effects of tourniquet release and continuous
passive motion. J Bone Joint Surg Am. 1991 Aug;73(7):1037-40.
PubMed PMID: 1874765.
Bourne RB. Continuous passive motion improves active knee
flexion and shortens hospital stay but does not affect other
functional outcomes after knee arthroplasty. J Bone Joint Surg
Am. 2005 Nov;87(11):2594. PubMed PMID: 16264143.

University of Pennsylvania Department of Orthopaedic Surgery


2012-234 (Joints)

University of Pennsylvania Department of Orthopaedic Surgery


Question 234

University of Pennsylvania Department of Orthopaedic Surgery


Question 234

In any patient with chronic pain after joint replacement,


infection should always be ruled out before other diagnoses
are explored. In this case, the patient’s ESR, CRP, and bone
scan are negative, making infection, insufficiency fracture,
and aseptic loosening less likely. Pain from anterior
iliopsoas impingement is believed to be related to a
prominent or malpositioned acetabular component, retained
cement, or excessively long screws. These patients usually
have persistent groin pain that is exacerbated by stair
climbing, getting into and out of bed, rising from a seated
position, and entering and exiting cars. The differential
diagnosis includes infection, component loosening, occult
fracture, or referred lumbar spine pathology. CT is useful in
measuring acetabular component version and to detect the
amount of the acetabular component that is uncovered
anteriorly. The ideal position of the cup is 45 degrees of
abduction and 15 degrees of anteversion. In this case, CT
scan has shown that the cup is retroverted and likely
uncovered anteriorly, resulting in impingement. Nonsurgical
treatment consists of rest, NSAIDs, and physical therapy.
Steroid injection has not been found to be successful. In
Answer = 5 most cases, surgical intervention is required. Removal of
impinging cement or screws can be definitive treatment.
Tendon release has also proven to be successful. Acetabular
revision is recommended when preoperative imagine shows
that the acetabular component protrudes in front of the
anterior bony acetabular rim.

University of Pennsylvania Department of Orthopaedic Surgery


2012-246 (Joints)
Which type of cells has been implicated in the
process shown in Figure 246?

1. Monocytes
2. Histiocytes
3. Leukocytes
4. Neutrophils
5. Macrophages

University of Pennsylvania Department of Orthopaedic Surgery


University of Pennsylvania Department of Orthopaedic Surgery
Hip and Knee
Question 246 Reconstruction

Which type of cells has been implicated in the


process shown in Figure 246?

1. Monocytes
2. Histiocytes
3. Leukocytes
4. Neutrophils
5. Macrophages

University of Pennsylvania Department of Orthopaedic Surgery


Hip and Knee
Question 246
Reconstruction
Explanation
THA is one of the most commonly performed and successful elective orthopaedic procedures.
However, numerous failure mechanisms limit the long-term success including aseptic
osteolysis, aseptic loosening, infection, and implant instability. Aseptic osteolysis and
subsequent implant failure occur because of a chronic inflammatory response to implant-
derived wear particles. To reduce particulate debris and their consequences, implants have
had numerous design modifications including high-molecular-weight polyethylene sockets
and noncemented implants that rely on bone ingrowth for fixation. Surgical techniques have
improved cementation with the use of medullary plugs, cement guns, lavage of the canal,
pressurization, centralization of the stem, and reduction in cement porosity. Despite these
advances, aseptic osteolysis continues to limit implant longevity. Numerous proinflammatory
cytokines, such as IL-1, IL-6, TNF-alpha, and prostaglandin E2, have proosteoclastogenic
effects in response to implant-derived wear particles. However, none of these cytokines
represents a final common pathway for the process of particle-induced osteoclast
differentiation and maturation. Macrophages are the key cells related to osteolysis at the
bone-implant interface. (n.b., macrophages are differentiated monocytes).
1.Ren PG, Irani A, Huang Z, Ma T, Biswal S, Goodman SB. Continuous infusion of UHMWPE particles
induces increased bone macrophages and osteolysis. Clin Orthop Relat Res. 2011 Jan;469(1):113-22.
PubMed PMID: 21042895.
2. Holt G, Murnaghan C, Reilly J, Meek RM. The biology of aseptic osteolysis. Clin Orthop Relat Res. 2007
Jul;460:240-52. Review. PubMed PMID: 17620815.

University of Pennsylvania Department of Orthopaedic Surgery


2012-254 (Joints)

Figure 254 is the radiograph of a 32-year-old woman treated


with high-dose steroids for a flare of systemic lupus
erythematous. The most appropriate surgical treatment for
the avascular necrosis lesion would be?

1. core decompression.
2. total hip arthroplasty.
3. bipolar hemiarthroplasty.
4. vascularized fibula grafting.
5. injection of platelet-rich plasma.
Images
Answer

• Question 254
Figure 254 is the radiograph of a 32-year-old woman treated
with high-dose steroids for a flare of systemic lupus
erythematous. The most appropriate surgical treatment for
the avascular necrosis lesion would be?

1. core decompression.
2. total hip arthroplasty.
3. bipolar hemiarthroplasty.
4. vascularized fibula grafting.
5. injection of platelet-rich plasma.
Explanation
The Xray demonstrates avascular necrosis with femoral head collapse and degenerative
changes of the acetabulum (Steinberg Stage V). Core decompression and vascularized
fibular grafting are only indicated for pre-collapse AVN (eliminate answers 1 and 4). PRP
is not a treatment option for collapsed AVN and bipolar hemiarthroplasty would be
contraindicated in a patient with acetabular changes (eliminate answers 3 and 5).
Therefore, the correct answer is a total hip arthroplasty.

•RECOMMENDED READINGS:
Mont MA, Ragland PS, Parvizi J. Surgical treatment of osteonecrosis of the hip. Instr
Course Lect. 2006;55:167-72. Review. PubMed PMID: 16958449.

Mont MA, Zywiel MG, Marker DR, McGrath MS, Delanois RE. The natural history of
untreated asymptomatic osteonecrosis of the femoral head: a systematic literature
review. J Bone Joint Surg Am. 2010 Sep 15;92(12):2165-70. Review. PubMed PMID:
20844158.
2012-263 (Joints)

The elution of antibiotics from a cement spacer is increased by?


 
1. vacuum mixing.
2. increased cement porosity.
3. the use of a static spacer.
4. the use of vancomycin alone.
5. the use of rifampin in combination with other antibiotics.

University of Pennsylvania Department of Orthopaedic Surgery


Question 263 – Preferred Response

The elution of antibiotics from a cement spacer is increased by?


 
1. vacuum mixing.
2. increased cement porosity.
3. the use of a static spacer.
4. the use of vancomycin alone.
5. the use of rifampin in combination with other antibiotics.

University of Pennsylvania Department of Orthopaedic Surgery


Question 263 - Explanation
This is a question if regarding some of the basic properties of antibiotic loaded cement.
The important concepts to know are as follows:
1.Antibiotic loaded cement increases local concentration of the antibiotic decreasing
systemic side effects while attaining a high concentration locally.
2.Antibiotics used must be thermostable and water soluble to avoid deterioration during
the exothermic polymerization reaction and to allow for diffusion after cement is set.
3.The elution of antibiotic is proportional to pore size and surface area with some other
character as well. Some notes here tobra and vanc together increase antibiotic elution.
Vacuum mixing decreases pore size and thus elution and hand mixing decreases
homogeneity and as a result elution.

This is a somewhat intuitive concept the more difficult question might be the same list
with tobra and van together listed as an option which would be the correct answer if pore
size were to be omitted.
RECOMMENDED READINGS:
Joseph TN, Chen AL, Di Cesare PE. Use of antibiotic-impregnated cement in total joint arthroplasty. J Am Acad Orthop Surg. 2003 Jan-Feb;11(1):38-47.
Review. PubMed PMID: 12699370.
Cui Q, Mihalko WM, Shields JS, Ries M, Saleh KJ. Antibiotic-impregnated cement spacers for the
treatment of infection associated with total hip or knee arthroplasty. J Bone Joint Surg Am. 2007
Apr;89(4):871-82. Review. PubMed PMID: 17403814.
Stevens CM, Tetsworth KD, Calhoun JH, Mader JT. An articulated antibiotic spacer used for infected total knee arthroplasty: a comparative in vitro elution
study of Simplex and Palacos bone cements. J Orthop Res. 2005 Jan;23(1):27-33. PubMed PMID: 15607871.
 

University of Pennsylvania Department of Orthopaedic Surgery


2012-270 (Joints)

• Figure 270 is the radiograph of a 55-year-old veteran who developed


avascular necrosis after a traumatic hip dislocation. He was treated with
hemiarthroplasty 10 years ago and also has posttraumatic stress
disorder and chronic pain. He has had multiple spinal surgeries and
takes 30 mg of methadone daily. He now has severe groin pain and is
unable to ambulate. Laboratory studies showed a C-reactive protein
level of 0.2 mg/L (reference range, 0-3 mg/L), erythrocyte
sedimentation rate of 50 mm/h (reference range, 0-20 mm/h), hip
aspiration of 500/mm3 white blood cell count, 50% polynucleated cells,
30% monocytes, and 20% lymphocytes. What is the most likely cause of
his hip pain?

1. Infection
2. Osteolysis
3. Acetabular protrusio
4. Loosening of implant
5. Complex regional pain syndrome
2012-270
2012-270

• Figure 270 is the radiograph of a 55-year-old veteran who developed


avascular necrosis after a traumatic hip dislocation. He was treated with
hemiarthroplasty 10 years ago and also has posttraumatic stress
disorder and chronic pain. He has had multiple spinal surgeries and
takes 30 mg of methadone daily. He now has severe groin pain and is
unable to ambulate. Laboratory studies showed a C-reactive protein
level of 0.2 mg/L (reference range, 0-3 mg/L), erythrocyte
sedimentation rate of 50 mm/h (reference range, 0-20 mm/h), hip
aspiration of 500/mm3 white blood cell count, 50% polynucleated cells,
30% monocytes, and 20% lymphocytes. What is the most likely cause of
his hip pain?

1. Infection
2. Osteolysis
3. Acetabular protrusio
4. Loosening of implant
5. Complex regional pain syndrome
2012-270

• Protrusio: medial displacement of femoral head medial


to ilioischial (Kohler’s) line
• No radiolucent lines on Xray suggesting osteolysis/loosening
• Aspiration negative for infection

• THA superior to bipolar hemiarthroplasty for AVN


– Better hip function after THA
– Less groin and buttock pain after THA
• Groin pain common indication for revision after hemi
– Articular cartilage degeneration
– Loosening of prosthesis in proximal femur
– Infection
• Hemiarthroplasty should be avoided in younger, active
patients -> increased risk of acetabular erosion
Question 20

Figures 20a and 20b are the radiograph and MRI scan of a 58-year-old man
who had total hip arthroplasty 3 years ago. His hip has been increasingly
painful for 6 months. Laboratory studies show an erythrocyte sedimentation
rate of 24 mm/h (reference range [rr], 0-20 mm/h) and a C-reactive protein
level of 0.3 mg/L (rr, 0.08-3.1 mg/L). In Figure 20b, which abnormality is
indicated by the arrows?
•1. Infection
•2. Malignancy
•3. Pseudotumor
•4. Polyethylene debris
•5. Heterotopic ossification
Question 20
Question 20

Figures 20a and 20b are the radiograph and MRI scan of a 58-year-old man
who had total hip arthroplasty 3 years ago. His hip has been increasingly
painful for 6 months. Laboratory studies show an erythrocyte sedimentation
rate of 24 mm/h (reference range [rr], 0-20 mm/h) and a C-reactive protein
level of 0.3 mg/L (rr, 0.08-3.1 mg/L). In Figure 20b, which abnormality is
indicated by the arrows?
•1. Infection
•2. Malignancy
•3. Pseudotumor
•4. Polyethylene debris
•5. Heterotopic ossification
Question 20

The radiograph shows a metal on metal hip implant. Although pseudotumors


can occur with any type of hip prosthesis, recent literature associates them
most frequently with metal on metal implants. A pseudotumor is neither
infectious nor neoplastic.

Daniel J, Holland J, Quigley L, Sprague S, Bhandari M. Pseudotumors


associated with total hip arthroplasty. J Bone Joint Surg Am. 2012 Jan
4;94(1):86-93. Review. PubMed PMID: 22218386.

Hart AJ, Satchithananda K, Liddle AD, Sabah SA, McRobbie D, Henckel J, Cobb
JP, Skinner JA, Mitchell AW. Pseudotumors in association with well-functioning
metal-on-metal hip prostheses: a case-control study using three-dimensional
computed tomography and magnetic resonance imaging. J Bone Joint Surg
Am. 2012 Feb 15;94(4):317-25. PubMed PMID: 22336970.
Arthroplasty

31

• Which population is least likely to receive total joint arthroplasty?

• 1. Black men
• 2. Black women
• 3. White men
• 4. White women
• 5. Hispanic men
31

• Which population is least likely to receive total joint arthroplasty?

• 1. Black men
• 2. Black women
• 3. White men
• 4. White women
• 5. Hispanic men
31
• The use of knee arthroplasty varies according to sex and race or ethnic group, with lower
rates among men, blacks, and Hispanics. The differences between the sexes have been
attributed to the higher rate of osteoarthritis among women. However, since rates of
osteoarthritis are generally higher among blacks and Hispanics than among whites, the
possibility of racial barriers must be considered.

• Skinner et al analyzed a total of 430,726 knee arthroplasties that were reported in the
Medicare claims data from 1998 through 2000.
• Among women, the national rates were higher for whites (5.97 procedures per 1000
women) than for Hispanics (5.37 per 1000) and blacks (4.84 per 1000) (P<0.001).

• Among men, the gap was more pronounced: the rate for whites (4.82 procedures
per 1000 men) was higher than that for Hispanics (3.46 per 1000) and more than
double the rate for blacks (1.84 per 1000, P<0.001).
• The rates were significantly lower for black men than for non-Hispanic white men in nearly
every region of the country (P<0.05).

• RECOMMENDED READINGS
• Skinner J, Weinstein JN, Sporer SM, Wennberg JE. Racial, ethnic, and geographic disparities in rates of knee arthroplasty among
Medicare patients. N Engl J Med. 2003 Oct 2;349(14):1350-9. PubMed PMID: 14523144.
• Nelson CL. Disparities in orthopaedic surgical intervention. J Am Acad Orthop Surg. 2007;15 Suppl 1:S13-7. PubMed PMID:
17766783.
Question 44

A 70-year-old healthy man had total knee arthroplasty 18 years ago, and it

now is painful. Radiographs reveal aseptic loosening and the range of motion

before surgery is 15 to 85 degrees. The strongest indication for performing a

tibial tubercle osteotomy to aid in exposure in his knee would be

1. patella baja.

2. nonresurfaced patella.

3. isolated femoral revision.

4. noncemented tibial component.

5. previous use of the quadriceps turn-down technique.

University of Pennsylvania Department of Orthopaedic Surgery


Question 44

A 70-year-old healthy man had total knee arthroplasty 18 years ago, and

it now is painful. Radiographs reveal aseptic loosening and the range of

motion before surgery is 15 to 85 degrees. The strongest indication for

performing a tibial tubercle osteotomy to aid in exposure in his knee

would be

1. patella baja.

2. nonresurfaced patella.

3. isolated femoral revision.

4. noncemented tibial component.

5. previous use of the quadriceps turn-down technique.


University of Pennsylvania Department of Orthopaedic Surgery
• Options for exposure in revision TKA: dissection of scar,
quadriceps snip, tibial tubercle osteotomy, medial epicondylar
osteotomy
• Goal in exposure is to avoid patellar tendon disruption and
skin necrosis
• TTO: expose proximal 10cm of tibial, cut medial to lateral, cut
should be left incomplete on the lateral side leaving the
periosteum and muscles attached, 2-3 wires passed around
the lateral edge of the tibial tubercle and back into the tibial
crest angled down 45 degrees to pull the osteotomy distally
• TTO allows the tibial shaft to be accessed for cement removal
and reduces tension on the extensor mechanism
• Decision to use tibial tubercle osteotomy based on whether
the patella could not be retracted with the knee bent at 90
degrees without risking patellar tendon avulsion

Tibial tubercle osteotomy in revision TKA allows for access to the tibial canal
whilst decreasing stress on the extensor mechanism and should be used in
cases where there is risk of patella tendon disruption with retraction in order
to gain access to the tibia
RECOMMENDED READINGS

Whiteside LA. Exposure in difficult total knee arthroplasty using tibial tubercle osteotomy. Clin Orthop Relat Res. 1995 Dec;(321):32-5. PubMed PMID:

7497683.

Younger AS, Duncan CP, Masri BA. Surgical exposures in revision total knee arthroplasty. J Am Acad Orthop Surg. 1998 Jan-Feb;6(1):55-64. Review.

PubMed PMID: 9692941.

Mendes MW, Caldwell P, Jiranek WA. The results of tibial tubercle osteotomy for revision total knee arthroplasty. J Arthroplasty. 2004 Feb;19(2):167-74.

PubMed PMID: 14973859. University of Pennsylvania Department of Orthopaedic Surgery


HIP & KNEE
RECONSTRUCTION 50
Figure 50 is the radiograph of a 45-year-old man who has avascular
necrosis of the hip attributable to his sickle cell anemia. He is
scheduled for total hip arthroplasty. To prevent the most likely
intrasurgical technical complication, particular attention should be
directed toward

• 1. dislocating the hip.


• 2. preparing the femur.
• 3. reaming the acetabulum.
• 4. inserting the acetabular screws.
• 5. cutting the short external rotators.
50
50
Figure 50 is the radiograph of a 45-year-old man who has
avascular necrosis of the hip attributable to his sickle cell
anemia. He is scheduled for total hip arthroplasty. To prevent
the most likely intrasurgical technical complication, particular
attention should be directed toward

• 1. dislocating the hip.


• 2. preparing the femur.
• 3. reaming the acetabulum.
• 4. inserting the acetabular screws.
• 5. cutting the short external rotators.
50
• The most common intraop complication in THA in
patients with SCA is preparation of the femur due to
widening of the medullary canal, thinning of the
trabeculae and cortices, and areas of dense bony
sclerosis.
• Marrow hyperplasia occurs in the setting of chronic anemia which
stimulates erythropoiesis, which subsequently leads to widening of the
marrow canal and thinning of the trabeculae. Additionally, repeated
episodes of marrow infarction result in patchy areas of sclerosis with
canal obliteration. These factors result in the femur being at high risk
for perforation with reaming and prosthesis fitting1,2. Rates of femoral
shaft perforation and fracture range from 4.9-18.2%1.
• The acetabulum (option 3) also prone to fx during implantation of
cementless component d/t marrow hyperplasia and sclerosis, but is
considerably less common than femur perforation1,2.
• Dislocation (option 1) can also be more difficult, particularly in the Demonstrating
setting of acetabular protrusio, and sometimes a preliminary intramedullary
osteotomy of femoral neck and/or neck may be required1,2. sclerosis,
marrow
• 1. Jeong GK, J Am Acad Orthop Surg 2005 hyperplasia and
• 2. Hernigou P, Clin Orthop Relat Res 2008 cortical
thickening1
Question 59

• A 63-year-old woman with rheumatoid arthritis is


undergoing a knee arthroplasty. Her rheumatoid
arthritis has been well controlled with methotrexate,
etanercept, and naproxen. Which medication-related
instructions should be followed 7 days before surgery?

1. Continue all medications


2. Discontinue naproxen
3. Discontinue naproxen and etanercept
4. Discontinue naproxen and methotrexate
5. Discontinue naproxen, etanercept, and methotrexate
Question 59

• A 63-year-old woman with rheumatoid arthritis is


undergoing a knee arthroplasty. Her rheumatoid
arthritis has been well controlled with methotrexate,
etanercept, and naproxen. Which medication-related
instructions should be followed 7 days before surgery?

1. Continue all medications


2. Discontinue naproxen
3. Discontinue naproxen and etanercept
4. Discontinue naproxen and methotrexate
5. Discontinue naproxen, etanercept, and methotrexate
NSAIDs should be discontinued 5 half-lives before surgery
(naproxen half-life: 12-17 hours); ASA discontinued 7-10 days
before surgery; etanercept discontinued for 1 week before
surgery; methotrexate may be continued perioperatively in those
without kidney, lung or liver disease, non-elderly, non-diabetics
(otherwise, consider holding 1-2 doses if undergoing moderate to
intensive procedure)

Howe CR, Gardner GC, Kadel NJ. Perioperative medication management for the patient with
rheumatoid arthritis. J Am Acad Orthop Surg. 2006 Sep;14(9):544-51. Review. PubMed
PMID: 16959892.
Giles JT, Bartlett SJ, Gelber AC, Nanda S, Fontaine K, Ruffing V, Bathon JM. Tumor necrosis
factor inhibitor therapy and risk of serious postoperative orthopedic infection in rheumatoid
arthritis. ArthritisRheum. 2006 Apr 15;55(2):333-7. PubMed PMID: 16583385.
Perhala RS, Wilke WS, Clough JD, Segal AM. Local infectious complications following large
joint replacement in rheumatoid arthritis patients treated with methotrexate versus those not
treated with methotrexate. Arthritis Rheum. 1991Feb;34(2):146-52. PubMed PMID: 1994911.
Question 76 Hip and Knee
Reconstruction
Figures 76a through 76c are the anteroposterior and lateral radiographs and bone
scan of a 66-year-old man with type I diabetes mellitus who had revision right total
knee arthroplasty for aseptic loosening 3 years ago. He has pain over the proximal
tibia with startup and at the end of the day. He has difficulty walking on level
ground.
Laboratory studies reveal an erythrocyte sedimentation rate of 5 mm/h (reference
range [rr], 0-20 mm/h) and C-reactive protein of <3.0 mg/L (rr, 0.08-3.1 mg/L).
Synovial fluid has 389 nucleated cells with 11% neutrophils and cultures are
negative.
What is the most likely failure mechanism for this revision total knee arthroplasty?
1. Unrecognized fungal infection
2. Improper component alignment
3. Posterior cruciate ligament insufficiency
4. Aseptic loosening because of inadequate diaphyseal fixation
5. Aseptic loosening because of inadequate metaphyseal fixation
Question 76 Hip and Knee
Reconstruction
Question 76 Hip and Knee
Reconstruction
Figures 76a through 76c are the anteroposterior and lateral radiographs and bone
scan of a 66-year-old man with type I diabetes mellitus who had revision right total
knee arthroplasty for aseptic loosening 3 years ago. He has pain over the proximal
tibia with startup and at the end of the day. He has difficulty walking on level
ground.
Laboratory studies reveal an erythrocyte sedimentation rate of 5 mm/h (reference
range [rr], 0-20 mm/h) and C-reactive protein of <3.0 mg/L (rr, 0.08-3.1 mg/L).
Synovial fluid has 389 nucleated cells with 11% neutrophils and cultures are
negative.
What is the most likely failure mechanism for this revision total knee arthroplasty?
1. Unrecognized fungal infection
2. Improper component alignment
3. Posterior cruciate ligament insufficiency
4. Aseptic loosening because of inadequate diaphyseal fixation
5. Aseptic loosening because of inadequate metaphyseal fixation
Question 76 Hip and Knee
Reconstruction
In this case, the patient has mechanical symptoms and no objective signs
of infection. The bone scan indicates metaphaseal involvement which
correlates to the area of pain. The patient underwent revision surgery
with an uncemented tibial stem. In revision surgery, metaphaseal
fixation can be achieved by the use of stems which transfer stress
distally in the tibia. Other techniques for greater amounts of
metaphaseal bone loss include metaphaseal cementation, allograft,
trabecullar metal forms, and metaphyseal sleeves. This patient likely
had inadequate metaphaseal fixation which is leading to the
mechanical symptoms described in the scenerio.

In the absence of infection, inadequate metaphaseal fixation after revision


surgery is a common failure mechanism.

RECOMMENDED READINGS
Haidukewych GJ, Hanssen A, Jones RD. Metaphyseal fixation in revision total knee arthroplasty: indications and techniques. J Am Acad Orthop Surg. 2011 Jun;19(6):311-
8. Review. PubMed PMID: 21628642.
Bush JL, Wilson JB, Vail TP. Management of bone loss in revision total knee arthroplasty. Clin Orthop Relat Res. 2006 Nov;452:186-92. Review. PubMed PMID: 16906109.
Joints

98. When templating THA, which figure reveals the


best recreation of the proper biomechanics of the
hip joint, assuming that the right leg is 5mm shorter
than the left?

1. 98a
2. 98b
3. 98c
4. 98d
5. 98e

University of Pennsylvania Department of Orthopaedic Surgery


98a 98b 98c 98d 98e

University of Pennsylvania Department of Orthopaedic Surgery


Joints

98. When templating THA, which figure reveals the


best recreation of the proper biomechanics of the
hip joint, assuming that the right leg is 5mm shorter
than the left?

1. 98a
2. 98b
3. 98c
4. 98d
5. 98e

University of Pennsylvania Department of Orthopaedic Surgery


Explanation:
Merle C, Waldstein W, Pegg E, Streit MR, Gotterbarm T, Aldinger
PR, Murray DW, Gill HS. Femoral offset is underestimated on
anteroposterior radiographs of the pelvis but accurately assessed
on anteroposterior radiographs of the hip. J Bone Joint Surg Br.
2012 Apr;94(4):477-82.
-AP of the hip is superior to AP of the pelvis for measuring
femoral offset

Della Valle AG, Padgett DE, Salvati EA. Preoperative planning for
primary total hip arthroplasty. J Am
Acad Orthop Surg. 2005 Nov;13(7):455-62. Review.
-Cup template: close to teardrop to reduce removal of
subchondral bone and restore hip center of rotation
-Femoral template: optimize limb length and femoral
offset to improve biomechanics
-Limb length change: vertical distance from center of
rotation of femoral component and that of the acetabular
component

Testable concept: Lengthening a limb s/p THA


involves adjustment of the femoral component,
placing the femoral template center of rotation
superior to that of the acetabulum
University of Pennsylvania Department of Orthopaedic Surgery
Arthroplasty

105: Internal rotation of the femoral component can cause patella maltracking by

1. increasing the Q angle.

2. increasing the medial-directed force vector on the patella.

3. producing valgus malalignment.

4. tightening of the lateral retinaculum.

5. overstuffing the patellofemoral compartment.


105: Internal rotation of the femoral component can cause patella maltracking by

1. increasing the Q angle.

2. increasing the medial-directed force vector on the patella.

3. producing valgus malalignment.

4. tightening of the lateral retinaculum.

5. overstuffing the patellofemoral compartment.


Explanation
The Q-angle is the angle between a line joining
the ASIS and the patella (direction of quad
pull) and the line from the tibial tuberosity
through the patella (direction of patella tendon
pull). Internal rotation of the femoral
component pulls the patella medial.

- Rhoads DD, Noble PC, Reuben JD, Tullos HS. The effect of femoral component position on the
kinematics of total knee arthroplasty. Clin Orthop Relat Res. 1993 Jan;(286):122-9. PubMed

PMID: 8425333.

- Malo M, Vince KG. The unstable patella after total knee arthroplasty: etiology, prevention, and
management. J Am Acad Orthop Surg. 2003 Sep-Oct;11(5):364-71. Review. PubMed PMID:

14565758

.
Question 108
• A 70-year-old man with osteoarthrosis is
scheduled to undergo total knee arthroplasty. He
inquires about patellar resurfacing. He should be
told that a potential advantage of having the
patella resurfaced as opposed to leaving the
patella unresurfaced is:
1. increased extensor strength.
2. lower risk for patellar fracture.
3. lower risk for requiring reoperation.
4. lower risk for patellar subluxation.
5. higher chance of achieving desirable range of
motion.
Question 108
• A 70-year-old man with osteoarthrosis is
scheduled to undergo total knee arthroplasty. He
inquires about patellar resurfacing. He should be
told that a potential advantage of having the
patella resurfaced as opposed to leaving the
patella unresurfaced is:
1. increased extensor strength.
2. lower risk for patellar fracture.
3. lower risk for requiring reoperation.
4. lower risk for patellar subluxation.
5. higher chance of achieving desirable range of
motion.
Explanation

• Non-resurfacing of the patellar during total


knee arthroplasty has been associated with
greater incidence of anterior knee pain and a
documented 8.7% rate of secondary
resurfacing of the patella.

• RECOMMENDED READINGS:
• Meneghini RM. Should the patella be resurfaced in primary total knee arthroplasty?
An evidence-based analysis. J Arthroplasty. 2008 Oct;23(7 Suppl):11-4. Epub 2008
Aug 12. Review. PubMed PMID: 18701250.
• Parvizi J, Rapuri VR, Saleh KJ, Kuskowski MA, Sharkey PF, Mont MA. Failure to
resurface the patella during total knee arthroplasty may result in more knee pain and
secondary surgery. Clin Orthop Relat Res. 2005 Sep;438:191-6. PubMed PMID:
16131890.
Question 121
Figures 121a and 121b are the current radiographs of a 39-year-old woman who had
left total hip arthroplasty 1 year ago. She is experiencing squeaking from the left hip
while ambulating. Which factor most likely contributes to her symptoms?
1. Activity level
2. Surgical approach
3. Component design
4. Component loosening
5. Component positioning

University of Pennsylvania Department of Orthopaedic Surgery


Question 121
Figures 121a and 121b are the current radiographs of a 39-year-old woman who had
left total hip arthroplasty 1 year ago. She is experiencing squeaking from the left hip
while ambulating. Which factor most likely contributes to her symptoms?
1. Activity level
2. Surgical approach
3. Component design
4. Component loosening
5. Component positioning
PREFERRED RESPONSE: 5

University of Pennsylvania Department of Orthopaedic Surgery


Question 121
Figures 121a and 121b are the current radiographs of a 39-year-old woman who had
left total hip arthroplasty 1 year ago. She is experiencing squeaking from the left hip
while ambulating. Which factor most likely contributes to her symptoms? PR:
Component positioning

This study found that squeaking appeared immediately in the nonlubricated


conditions with the presence of ceramic on ceramic wear, specifically stripe wear. As
with other types of wear, the factor most responsible for stripe wear is component
malpostion.

University of Pennsylvania Department of Orthopaedic Surgery


Question 123
A 68-year-old woman fell and sustained a displaced femoral neck fracture.
She is a community ambulator and enjoys playing tennis weekly. Which
treatment will provide her with the best hip function?
1. Hip resurfacing
2. Hemiarthroplasty
3. Total hip arthroplasty
4. Internal fixation with cannulated screws
5. Internal fixation with a sliding hip screw and an antirotation screw

University of Pennsylvania Department of Orthopaedic Surgery


Question 123
A 68-year-old woman fell and sustained a displaced femoral neck fracture.
She is a community ambulator and enjoys playing tennis weekly. Which
treatment will provide her with the best hip function?
1. Hip resurfacing
2. Hemiarthroplasty
3. Total hip arthroplasty
4. Internal fixation with cannulated screws
5. Internal fixation with a sliding hip screw and an antirotation screw

PREFERRED RESPONSE: 3

University of Pennsylvania Department of Orthopaedic Surgery


Question 123
A 68-year-old woman fell and sustained a displaced femoral neck fracture.
She is a community ambulator and enjoys playing tennis weekly. Which
treatment will provide her with the best hip function?
PR: Total hip arthroplasty

In this study, the seven- to ten-year results (of 81 pts) indicate longer
survival of patients treated by THR. There was also a trend towards better
function, less pain and fewer re-operations.

University of Pennsylvania Department of Orthopaedic Surgery


Arthoplasty

Question 136

What is the optimal treatment for a Vancouver type B2


fracture in a healthy patient?

1. Retain the stem and fracture fixation with cortical strut


graft and cables
2. Revision to a proximal femoral-replacing stem
3. Revision to a long porous-coated stem and cable fixation
4. Revision to a long cemented stem bypassing the fracture
site
5. Revision to a proximally coated stem and open reduction
and internal fixation of the fracture

University of Pennsylvania Department of Orthopaedic Surgery


Question 136 – Preferred Response

What is the optimal treatment for a Vancouver type B2


fracture in a healthy patient?

1. Retain the stem and fracture fixation with cortical strut


graft and cables
2. Revision to a proximal femoral-replacing stem
3. Revision to a long porous-coated stem and cable fixation
4. Revision to a long cemented stem bypassing the fracture
site
5. Revision to a proximally coated stem and open reduction
and internal fixation of the fracture

University of Pennsylvania Department of Orthopaedic Surgery


Question 136 - Explanation

Corten, et al. reported on using cemented stem with fracture fixation for Vancouver B2 in
elderly patients with limited life expectancy. 43% of their patients had died by 1 year follow-up.
They did report good outcomes on surviving patients. Advantages in this patient population
were quicker return to weight bearing and reduced costs of implants.

Mulay, et al. reported excellent results using a modular, distally-fixed uncemented prosthesis.

Springer, et al compared results of various femoral component revisions for periprosthetic


fractures and found best results in uncemented, extensively porous-coated stems

Treatment of Vancouver B2 periprosthetic hip fractures is via revision of femoral component to


a long porous-coated cementless stem to bypass the fracture with appropriate fracture fixation.
Corten K, Macdonald SJ, McCalden RW, Bourne RB, Naudie DD. Results of cemented femoral revisions for periprosthetic
femoral fractures in the elderly. J Arthroplasty. 2012 Feb;27(2):220-5. Epub 2011 Jul 12. PubMed PMID: 21752585.

Mulay S, Hassan T, Birtwistle S, Power R. Management of types B2 and B3 femoral periprosthetic fractures by a tapered,
fluted, and distally fixed stem. J Arthroplasty. 2005 Sep;20(6):751-6. PubMed PMID: 16139712.

Springer BD, Berry DJ, Lewallen DG. Treatment of periprosthetic femoral fractures following total hip arthroplasty with femoral
component revision. J Bone Joint Surg Am. 2003 Nov;85-A(11):2156-62. PubMed PMID: 14630846.

University of Pennsylvania Department of Orthopaedic Surgery


Arthroplasty
Question 153

• The failure of total hip arthroplasty using a


zirconium-ceramic femoral head as seen in
Figures 153a and 153b is most likely the
result of

• 1. infection.
• 2. aseptic loosening.
• 3. bony impingement.
• 4. material properties.
• 5. component alignment.
Question 153


Question 153

• The failure of total hip arthroplasty using a


zirconium-ceramic femoral head as seen in
Figures 153a and 153b is most likely the
result of

• 1. infection.
• 2. aseptic loosening.
• 3. bony impingement.
• 4. material properties.
• 5. component alignment.
Question 153

• The attached photos and question do not show any evidence of


aseptic loosening, infection, bony impingement or component
malalignment. Zirconium-ceramic heads have a track record of
head-fracture complications, as ceramic is a very brittle
biomaterial with a high Young’s modulus, low tensile strength and
poor crack resistance.

• Traina F, Tassinari E, De Fine M, Bordini B, Toni A. Revision of ceramic hip replacements for fracture
of a ceramic component: AAOS exhibit selection. J Bone Joint Surg Am. 2011 Dec 21;93(24):e147.
Review. PubMed PMID: 22258782.

• Hannouche D, Hamadouche M, Nizard R, Bizot P, Meunier A, Sedel L. Ceramics in total hip


replacement.Clin Orthop Relat Res. 2005 Jan;(430):62-71. Review. PubMed PMID: 15662305.
Hip and Knee Reconstruction

• Question 173
• A 57-year-old woman had right total knee arthroplasty for
varus gonarthrosis. Before surgery, her range of motion
was 5 to 110 degrees. At skin closure, her range of motion
was 0 to 120 degrees. Her range of motion at 10 weeks
after surgery is 0 to 70 degrees. What is the best next
treatment step?

• 1. Observation
• 2. Dynamic bracing
• 3. Manipulation under anesthesia
• 4. Revision with open adhesiolysis
• 5. Physical therapy with aggressive range of motion.
Hip and Knee Reconstruction

• Question 173
• A 57-year-old woman had right total knee arthroplasty for
varus gonarthrosis. Before surgery, her range of motion
was 5 to 110 degrees. At skin closure, her range of motion
was 0 to 120 degrees. Her range of motion at 10 weeks
after surgery is 0 to 70 degrees. What is the best next
treatment step?

• 1. Observation
• 2. Dynamic bracing
• 3. Manipulation under anesthesia
• 4. Revision with open adhesiolysis
• 5. Physical therapy with aggressive range of motion.
• Namba et al showed that flexion after TKA can be improved with both
early (<90 days) and late (>90 days) MUA. They did a retrospective
review that found 195 pts with MUA (102 early and 93 late). Both groups
had significant improvement in flexion as well as significant improvement
in pain. Early MUA improved from 68.4 degrees to 101.4 degrees. Late
MUA improved from 81.0 degrees to 98.0 degrees. However they found
that extension only improved in the early group.
• Keating et al also showed improvement in flexion after TKA. They showed
an average improvement in flexion of 35 degrees at 5 year follow up
following MUA. The MUA was performed at an average of 10 weeks in 113
TKAs.

• Namba RS, Inacio M. Early and late manipulation improve flexion after total knee arthroplasty. J Arthroplasty. 2007 Sep;22(6 Suppl 2):58-61. Epub
2007 Jul 26. PubMed PMID: 17823017.
• Keating EM, Ritter MA, Harty LD, Haas G, Meding JB, Faris PM, Berend ME. Manipulation after total knee arthroplasty. J Bone Joint Surg Am. 2007
Feb;89(2):282-6. PubMed PMID: 17272441.
Hip and Knee
Reconstruction
Question 189
Question 189
Question 189
Hip and Knee Recon

Question 197

• Figure 197 is the radiograph of a 62-year-old


woman who is seen in the emergency
department with a dislocated left THA. This is
her seventh dislocation during the last 3
months and she most recently had a liner
revision. What is the best next treatment
step?
– 1. Skeletal traction
– 2. Open reduction
– 3. Closed reduction
– 4. Component revision
– 5. Hip abduction orthosis

University of Pennsylvania Department of Orthopaedic Surgery


Question 197

University of Pennsylvania Department of Orthopaedic Surgery


Question 197

• Figure 197 is the radiograph of a 62-year-old


woman who is seen in the emergency
department with a dislocated left THA. This is
her seventh dislocation during the last 3
months and she most recently had a liner
revision. What is the best next treatment
step?
– 1. Skeletal traction
– 2. Open reduction
– 3. Closed reduction
– 4. Component revision
– 5. Hip abduction orthosis

University of Pennsylvania Department of Orthopaedic Surgery


Question 197

• Dislocation following THA is the second most common


cause for revision THA behind infection
• Ideal positioning of the acetabular component is 40
degrees of abduction and 15 degrees anteversion give
or take 10 degrees in each position
• 2/3 of early dislocations can be treated with closed
reduction and immobilization
• This is frequently discussed in joints conference by the
rule of 3. First hip dislocation gets a closed reduction
and abduction brace assuming radiographs
demonstrate properly placed components. Second
dislocation gets a closed reduction plus or minus CT
scan to assess component positioning. Third
dislocation warrants revision THA.

University of Pennsylvania Department of Orthopaedic Surgery


Question 214

• Figure 214 is the current radiograph of a 74-year-old


man who had right total hip arthroplasty 3 weeks ago.
He stumbled and has increasing pain with weight-
bearing activity. What is the best next treatment
step?

• 1. Revision
• 2. Resection arthroplasty
• 3. Routine follow-up at 3 months
• 4. Open reduction and internal fixation
• 5. Nonweight bearing activity for 6 weeks

University of Pennsylvania Department of Orthopaedic Surgery


Question 214

University of Pennsylvania Department of Orthopaedic Surgery


Question 214

• Figure 214 is the current radiograph of a 74-year-old


man who had right total hip arthroplasty 3 weeks ago.
He stumbled and has increasing pain with weight-
bearing activity. What is the best next treatment
step?

• 1. Revision
• 2. Resection arthroplasty
• 3. Routine follow-up at 3 months
• 4. Open reduction and internal fixation
• 5. Nonweight bearing activity for 6 weeks

University of Pennsylvania Department of Orthopaedic Surgery


Question 214

• The patient has a periprosthetic fracture


through the calcar during the immediate
postoperative period with subsidence of the
femoral stem.
• Treatment of acute periprosthetic
fracture of the calcar is removal of the
stem, fixation of the fracture (usually
with cables), and revision of the femur
to a beaded fully coated stem
Duncan CP, Masri BA. Fractures of the femur after hip replacement. Instr Course Lect. 1995;44:293-304. Review. PubMed PMID:
7797866.
Springer BD, Berry DJ, Lewallen DG. Treatment of periprosthetic femoral fractures following total hip arthroplasty with femoral
component revision. J Bone Joint Surg Am. 2003 Nov;85-A(11):2156-62. PubMed PMID: 14630846.

University of Pennsylvania Department of Orthopaedic Surgery


234
• Figure 234a is the clinical photograph of an 82-year-old man who had left total knee
arthroplasty 1 year ago. He has difficulty with pain and stiffness and recently noted
swelling on the medial side. He had aspiration of the knee 1 month ago with a cell
count of 22,000/mm3 nucleated cells. Aerobic and anaerobic culture and gram stain
findings are negative. Laboratory studies reveal the erythrocyte sedimentation rate
and C-reactive protein are within defined limits. He is able to perform a straight-leg
• raise. Range of motion is 15 to 80 degrees. Anteroposterior and lateral radiographs
are shown in Figures 234b and 234c. What is the best next step?
• 1. An MRI scan to evaluate for possible vastus medialis oblique disruption
• 2. Physical therapy with biofeedback focusing on gentle range of motion
• 3. Reaspirate and send for aerobic, anaerobic, fungal, and acid fast bacilli cultures
• 4. Resection arthroplasty and placement of vancomycin and gentamicin cement
spacer
• 5. Revision total knee arthroplasty, elevation of joint line for flexion contracture,
repair of the
• extensor mechanism disruption
234
234
• Figure 234a is the clinical photograph of an 82-year-old man who had left total knee
arthroplasty 1 year ago. He has difficulty with pain and stiffness and recently noted
swelling on the medial side. He had aspiration of the knee 1 month ago with a cell
count of 22,000/mm3 nucleated cells. Aerobic and anaerobic culture and gram stain
findings are negative. Laboratory studies reveal the erythrocyte sedimentation rate
and C-reactive protein are within defined limits. He is able to perform a straight-leg
• raise. Range of motion is 15 to 80 degrees. Anteroposterior and lateral radiographs
are shown in Figures 234b and 234c. What is the best next step?
• 1. An MRI scan to evaluate for possible vastus medialis oblique disruption
• 2. Physical therapy with biofeedback focusing on gentle range of motion
• 3. Reaspirate and send for aerobic, anaerobic, fungal, and acid fast bacilli cultures
• 4. Resection arthroplasty and placement of vancomycin and gentamicin cement
spacer
• 5. Revision total knee arthroplasty, elevation of joint line for flexion contracture,
repair of the
• extensor mechanism disruption
234
• Phelan DM, Osmon DR, Keating MR, Hanssen AD. Delayed reimplantation
arthroplasty for candidal prosthetic joint infection: a report of 4 cases and review of
the literature. Clin Infect Dis. 2002 Apr 1;34(7):930-8. Epub 2002 Feb 26. Review.
PubMed PMID: 11880958.
• Azzam K, Parvizi J, Jungkind D, Hanssen A, Fehring T, Springer B, Bozic K, Della
Valle C, Pulido L, Barrack R. Microbiological, clinical, and surgical features of fungal
prosthetic joint infections: a multiinstitutional experience. J Bone Joint Surg Am.
2009 Nov;91 Suppl 6:142-9. PubMed PMID: 19884422.

• Patient history and physical exam shows continued effusion with limitation
in range of motion, concerning for deep space infection, without proven
culture data
• Ability to perform straight leg raise = Less likely extensor mechanism
rupture
• Complete cultures, including Fungal, AFB, Anaerobic needed to rule out
deep infection
Question 253 (Joints/ Trauma)

• Figure 253 shows the fracture sustained by an otherwise


healthy 61-year-old man who was knocked down by an
automobile door that was suddenly opened as he was
riding his bicycle. Which treatment will most likely provide
him with the best long-term function?

• 1. Hemiarthroplasty
• 2. Total hip arthroplasty
• 3. Open reduction and internal fixation with a blade plate
• 4. Open reduction and internal fixation with a dynamic hip screw
• 5. Closed reduction and percutaneous cannulated screw fixation
Question 253 (Joints/ Trauma)
Question 253 (Joints/ Trauma)

• Figure 253 shows the fracture sustained by an otherwise


healthy 61-year-old man who was knocked down by an
automobile door that was suddenly opened as he was
riding his bicycle. Which treatment will most likely provide
him with the best long-term function?

• 1. Hemiarthroplasty
• 2. Total hip arthroplasty
• 3. Open reduction and internal fixation with a blade plate
• 4. Open reduction and internal fixation with a dynamic hip screw
• 5. Closed reduction and percutaneous cannulated screw fixation
Question 253 (Joints/ Trauma)
• This radiographs reveal an acute displaced femoral neck fracture in an otherwise
healthy and active (“riding bicycle”) elderly male with apparent acetabular arthrosis.

• They key to this questions stems from discerning which treatment options “will most
likely provide him with the best long-term function”…

• The treatment of displaced FNF in elderly patients has evolved. Typically, arthroplasty
procedures have a shortened duration of post-operative rehabilitation and also avoid the
problems related to fracture healing of the femoral head. Though hemiarthroplasty is an
option, this procedure is typically reserved for patients with no pre-existing arthrosis and
patients who are typically less physically active. Taking all of this into account, as
described by Lee et al (JBJS, 1998), the procedure that provides the best long-term
function in the acutely displaced femoral neck fracture is total hip arthroplasty.

• THA arthroplasty provides the best long-term function in the elderly population
with acute displaced femoral neck fractures.

• Lee BP, Berry DJ, Harmsen WS, Sim FH. Total hip arthroplasty for the treatment of an acute fracture of the femoral neck:
long-term results. J Bone Joint Surg Am. 1998 Jan;80(1):70-5. PubMed PMID: 9469311.
• Ricci WM, Langer JS, Leduc S, Streubel PN, Borrelli JJ. Total hip arthroplasty for acute displaced femoral neck fractures via the
posterior approach: a protocol to minimize hip dislocation risk. Hip Int. 2011 Jun 8;21(3):344-350. doi:
10.5301/HIP.2011.8401. PubMed PMID: 21698586.
Arthroplasty
Question 272

• Figures 272a through 272c are the current radiographs and CT


reconstruction scan of a 58-year-old woman who has increasing
pain with household ambulation. An intrasurgical video shows
discontinuity of the pelvic ring with stress applied. After
undergoing treatment as seen in Figure 272d, what is the most
likely complication?
• 1. Infection
• 2. Instability
• 3. Nonunion
• 4. Aseptic loosening
• 5. Periprosthetic fracture
Question 272
Question 272

• Figures 272a through 272c are the current radiographs and CT


reconstruction scan of a 58-year-old woman who has increasing
pain with household ambulation. An intrasurgical video shows
discontinuity of the pelvic ring with stress applied. After
undergoing treatment as seen in Figure 272d, what is the most
likely complication?
• 1. Infection
• 2. Instability
• 3. Nonunion
• 4. Aseptic loosening
• 5. Periprosthetic fracture
Question 272

• The radiographs and video show discontinuity of the pelvis with a


defect isolating the acetabulum from the superior pelvis.

• Taunton MJ, Fehring TK, Edwards P, Bernasek T, Holt GE, Christie MJ. Pelvic discontinuity treated
with custom triflange component: a reliable option. Clin Orthop Relat Res. 2012 Feb;470(2):428-
34. PubMed PMID: 21997785.
• Christie MJ, Barrington SA, Brinson MF, Ruhling ME, DeBoer DK. Bridging massive acetabular
defects with the triflange cup: 2- to 9-year results. Clin Orthop Relat Res. 2001 Dec;(393):216-27.
PubMed PMID: 11764351.
Question 5
During a total hip arthroplasty, the surgeon inadvertently injects
a bolus of bupivacaine into the femoral vein. The patient goes
into asystole. Which agent is the treatment of choice to correct
this situation?
1. Propranolol
2. Epinephrine
3. 20% fat emulsion
4. Norepinephrine bitartrate
5. Phenylephrine hydrochloride
Question 5
During a total hip arthroplasty, the surgeon inadvertently injects
a bolus of bupivacaine into the femoral vein. The patient goes
into asystole. Which agent is the treatment of choice to correct
this situation?
1. Propranolol
2. Epinephrine
3. 20% fat emulsion
4. Norepinephrine bitartrate
5. Phenylephrine hydrochloride
IV lipid infusions
•Bupivacaine is a potent depressant of electrical
conduction, which predisposes the heart to reentry
types of arrythmias
•IV lipid emulsion infusions increase the dose of
bupivacaine required to produce asystole in
rats/dogs and improve survival.
•Human case reports have shown successful
resuscitation with IV lipid emulsion infusion following
bupivacaine-induced cardiovascular collapse
Question 20
While performing primary total knee arthroplasty using a cruciate-
retaining knee implant, a surgeon notices an iatrogenic injury to the medial
collateral ligament (MCL) following femoral component preparation. The
injury is a saw cut at the level of the joint line with partial transection of
the MCL, resulting in valgus laxity. What is the best next step?
1. Convert to a posterior stabilized (PS) knee design.
2. Convert to a hinged knee design.
3. Repair the MCL using heavy sutures or suture anchors.
4. Repair the MCL using heavy sutures or suture anchors and brace the
patient postsurgically.
5. Repair the MCL using heavy sutures or suture anchors and convert to a
PS knee implant.
Question 20
While performing primary total knee arthroplasty using a cruciate-
retaining knee implant, a surgeon notices an iatrogenic injury to the medial
collateral ligament (MCL) following femoral component preparation. The
injury is a saw cut at the level of the joint line with partial transection of
the MCL, resulting in valgus laxity. What is the best next step?
1. Convert to a posterior stabilized (PS) knee design.
2. Convert to a hinged knee design.
3. Repair the MCL using heavy sutures or suture anchors.
4. Repair the MCL using heavy sutures or suture anchors and brace the
patient postsurgically.
5. Repair the MCL using heavy sutures or suture anchors and convert to a
PS knee implant.
Question 20
PREFERRED RESPONSE: 4
Intraoperative MCL disruption can be treated with primary repair (midsubstance)
or suture anchor fixation (avulsion from femur/tibia) with 6 weeks of bracing,
unlimited ROM. No patients in the mentioned study had coronal plane instability
at 0 or 30 degrees or required bracing past 6 weeks post-operatively. Placing a
prosthesis of increased constraint is not usually necessary.
RECOMMENDED READINGS
Lee GC, Lotke PA. Management of intraoperative medial collateral ligament injury
during TKA. Clin Orthop Relat Res. 2011 Jan;469(1):64-8. doi: 10.1007/s11999-010-
1502-6. PubMed PMID: 20686933; PubMed Central PMCID: PMC3008909.
Leopold SS, McStay C, Klafeta K, Jacobs JJ, Berger RA, Rosenberg AG. Primary
repair of intraoperative disruption of the medical collateral ligament during total
knee arthroplasty. J Bone Joint Surg Am. 2001 Jan;83-A(1):86-91. PubMed PMID:
11205863.
Question 30
An otherwise healthy 60-year-old woman has intermittent severe knee pain and
effusions 10 years after undergoing total knee arthroplasty. She denies recent
infections. Radiographs show normal alignment and no osteolysis. Examination reveals
a large effusion, and range of motion is 10 to 110 degrees. She has slight varus-valgus
laxity. Her C-reactive protein level is 11 mg/L (reference range [rr], 0.08-3.1 mg/L) and
her erythrocyte sedimentation rate is 40 mm/h (rr, 0-20 m/h). Aspiration of the knee
reveals a white blood cell count of 8000 and 95% neutrophils. Cultures are negative.
What is the best treatment option?
1. Observation
2. Open synovectomy
3. Arthroscopic synovectomy
4. Revision of all components
5. Removal of all components
Question 30
An otherwise healthy 60-year-old woman has intermittent severe knee pain and
effusions 10 years after undergoing total knee arthroplasty. She denies recent
infections. Radiographs show normal alignment and no osteolysis. Examination reveals
a large effusion, and range of motion is 10 to 110 degrees. She has slight varus-valgus
laxity. Her C-reactive protein level is 11 mg/L (reference range [rr], 0.08-3.1 mg/L) and
her erythrocyte sedimentation rate is 40 mm/h (rr, 0-20 m/h). Aspiration of the knee
reveals a white blood cell count of 8000 and 95% neutrophils. Cultures are negative.
What is the best treatment option?
1. Observation
2. Open synovectomy
3. Arthroscopic synovectomy
4. Revision of all components
5. Removal of all components
Prosthetic joint infection (PJI)
MEMORIZE THESE
Major criteria:
•Sinus tract communicating with the prosthesis
•Pathogen isolated by culture from at least two separate tissue or fluid samples obtained from the affected
prosthetic joint
Minor criteria:
•Elevated ESR/CRP
•Elevated WBC count (>1750/ml)
•Elevated PMNs
•Pus in the joint
•Isolation of a microorganism in one culture of periprosthetic tissue or fluid
•> 5 PMNs per high-power field in five high-power fields observed from histologic analysis of periprosthetic
tissue at 9400 magnification.

PJI exists if one major criteria or 4/6 minor criteria present


This patient has a PJI. Standard of care in the US (and for OITE purposes) is two-stage removal of
implant and revision

Parvizi et al. New definition for Periprosthetic Joint Infection, CORR, 2011.
Question 44
• When templating for total hip arthroplasty, which
image demonstrates the best recreation of the
proper biomechanics of the hip joint, assuming
that the patient’s left leg is 8 mm longer than the
right?
– 1. Figure 44a
– 2. Figure 44b
– 3. Figure 44c
– 4. Figure 44d
– 5. Figure 44e
Question 44
Question 44
• When templating for total hip arthroplasty, which
image demonstrates the best recreation of the
proper biomechanics of the hip joint, assuming
that the patient’s left leg is 8 mm longer than the
right?
– 1. Figure 44a
– 2. Figure 44b
– 3. Figure 44c
– 4. Figure 44d
– 5. Figure 44e
Question 44
• Figure 44a- Would maintain the same limb length
discrepancy
• Figure 44b- Would increase limb length by 8mm by
templating the new hip center to be 8mm superior to
the current hip center
• Figure 44c- Would lateralize the hip and change the
tension of the soft tissues but not change limb length
• Figure 44d- Would decrease limb length by 8mm to an
already shorter limb by templating the new hip center
to be 8mm inferior to the current hip center
• Figure 44e- Would change soft tissue tension as well as
lengthen the limb
Question 50
Figure 50 is the clinical photograph of a healthy and active 50-year-old man
who underwent total knee arthroplasty 10 weeks ago. Wound drainage,
which occurred for more than 1 week after the index procedure, was treated
with oral antibiotics and local wound care. He is now in the emergency
department and has had increasing pain and swelling around the knee for 3
days. What is the best next step?

•1. A 6-week course of intravenous antibiotics


•2. A 2-stage protocol
•3. A single-stage exchange
•4. Irrigation, debridement, and polyethylene exchange
•5. Chronic suppressive oral antibiotic therapy
Question 50

Figure 50
Question 50
Figure 50 is the clinical photograph of a healthy and active 50-year-old man
who underwent total knee arthroplasty 10 weeks ago. Wound drainage,
which occurred for more than 1 week after the index procedure, was treated
with oral antibiotics and local wound care. He is now in the emergency
department and has had increasing pain and swelling around the knee for 3
days. What is the best next step?

•1. A 6-week course of intravenous antibiotics


•2. A 2-stage protocol
•3. A single-stage exchange
•4. Irrigation, debridement, and polyethylene exchange
•5. Chronic suppressive oral antibiotic therapy
Question 50
Simple irrigation and debridement of periprosthetic infections has shown to have high failure rates.
The most effective method of treating periprosthetic infections after arthroplasty is a 2 stage protocol.
The first stage involves removal of the infected prosthesis and implantation of an antibiotic-
impregnated spacer. After this, the patient receives 6 weeks of antibiotics targeting the organism
grown from intraoperative cultures. The second stage involves removing the antibiotic spacer and
replacing it with a new prosthesis. Antibiotic cement is typically used for the new prosthesis. Using the
one stage protocol, where in the prosthesis is replaced immediately after removal, has been shown to
be less effective at eradicating infection.

RECOMMENDED READINGS
Parvizi J, Ghanem E, Menashe S, Barrack RL, Bauer TW. Periprosthetic infection: what are the diagnostic challenges? J Bone Joint Surg Am.
2006 Dec;88 Suppl 4:138-47. PubMed PMID: 17142443.
Koyonos L, Zmistowski B, Della Valle CJ, Parvizi J. Infection control rate of irrigation and débridement for periprosthetic joint infection. Clin
Orthop Relat Res. 2011 Nov;469(11):3043-8. doi: 10.1007/s11999-011-1910-2. PubMed PMID: 21553171; PubMed Central PMCID:
PMC3183205.
Question 59
•A patient is undergoing the second stage of a 2-stage
exchange for a previously infected total knee arthroplasty. The
infection has resolved. However, surgical exposure is difficult
to achieve with a patella baja, scarred patellar tendon, and
profuse cement in the proximal tibia. What is the best surgical
option?
1. Lateral release
2. Full quadricep turndown
3. Z lengthening of patellar tendon
4. Extended tibial tubercle osteotomy
5. Patella tendon detachment and subsequent reattachment with a
toothed screw at the time of closure
Question 59
•A patient is undergoing the second stage of a 2-stage
exchange for a previously infected total knee arthroplasty. The
infection has resolved. However, surgical exposure is difficult
to achieve with a patella baja, scarred patellar tendon, and
profuse cement in the proximal tibia. What is the best surgical
option?
1. Lateral release
2. Full quadricep turndown
3. Z lengthening of patellar tendon
4. Extended tibial tubercle osteotomy
5. Patella tendon detachment and subsequent reattachment with a
toothed screw at the time of closure
Question 59
• The listed preferred response is a lateral release procedure, but the suggested readings
seem to suggest an extended tibial tubercle osteotomy would be a better procedure
given the preexisting patellar malalignment. In TTO techniques, the distal release is
performed through bone, mobilization of the anterior structures is excellent, and repair
can be secured by bone to bone fixation, permitting early rehabilitation and restoration
of quadriceps excursion and strength. In contrast, with proximal exposures through soft
tissue, such as the VY quadricepsplasty or turndown, range of motion and resistance
exercises are delayed and extensor lag can occur. This osteotomy has been used
successfully in the treatment of severe fractures of the distal femur, proximal tibia, and
the patella, and has gained even wider use in the management of patellar
malalignment conditions, in which the tubercle can be elevated or medialized to
improve patellofemoral congruence and tracking

Mendes MW, Caldwell P, Jiranek WA. The results of tibial tubercle osteotomy for revision total knee
arthroplasty. J Arthroplasty. 2004 Feb;19(2):167-74. PubMed PMID: 14973859.
Adult Recon

Question 76

A patient has had several dislocations 12 months


after undergoing a seemingly successful revision
total hip arthroplasty. The arthroplasty was
performed after the patient experienced a failed
metal-on-metal bearing and abductor damage.
What is the most appropriate course of action?

1. Trochanteric advancement
2. Surgical repair of the abductors
3. Application of an abduction brace
4. Revision with constrained polyethylene liner
5. Revision with increased ball head size, length,
and offset
Question 76 – Preferred Response

A patient has had several dislocations 12 months


after undergoing a seemingly successful revision
total hip arthroplasty. The arthroplasty was
performed after the patient experienced a failed
metal-on-metal bearing and abductor damage.
What is the most appropriate course of action?

1. Trochanteric advancement
2. Surgical repair of the abductors
3. Application of an abduction brace
4. Revision with constrained polyethylene liner
5. Revision with increased ball head size, length,
and offset
Question 76 - Explanation

• Abductor insufficiency is the only “test/board” answer for use


of a constrained liner
• Metal-on-Metal can lead to extensive soft tissue injury
through ALVAL and development of pseudotumors and
abductor insufficiency is increased following M-o-M revision
• Outcomes following M-o-M revision are generally poor

RECOMMENDED READINGS
Sikes CV, Lai LP, Schreiber M, Mont MA, Jinnah RH, Seyler TM. Instability after total hip arthroplasty: treatment with large
femoral heads vs constrained liners. J Arthroplasty. 2008 Oct;23(7 Suppl):59-63. doi: 10.1016/j.arth.2008.06.032.
Review. PubMed PMID: 18922375.
Killampalli VV, Reading AD. Late instability of bilateral metal on metal hip resurfacings due to progressive local tissue effects.
Hip Int. 2009 Jul-Sep;19(3):287-91. PubMed PMID: 19876887.
Question 98
Based on the acetabular defect seen in Figures 98a through 98c,
•What is the best treatment
1. Impaction grafting
2. Modular head and polyethylene liner exchange
3. Reconstruction with acetabular reinforcement cage
4. Cementless reconstruction with a porous hemispherical shell
5. Cementless reconstruction with a porous cup and highly porous
augment
Figure 98a Figure 98b Figure 98c
Question 98
Based on the acetabular defect seen in Figures 98a through 98c,
What is the best treatment
1. Impaction grafting
2. Modular head and polyethylene liner exchange
3. Reconstruction with acetabular reinforcement cage
4. Cementless reconstruction with a porous hemispherical shell
5. Cementless reconstruction with a porous cup and highly porous
augment
Classification of acetabular bone
loss

Sheth et al.
Sheth NP, Nelson CL, Springer BD, Fehring TK, Paprosky WG. Acetabular bone loss in revision total hip arthroplasty: evaluation
and management. J Am Acad Orthop Surg. 2013 Mar;21(3):128-39. doi: 10.5435/ JAAOS-21-03-128. Review. PubMed PMID:
23457063.
Issack PS. Use of porous tantalum for acetabular reconstruction in revision hip arthroplasty. J Bone Joint Surg Am. 2013 Nov 6;95(21):1981-
7. doi: 10.2106/JBJS.L.01313. Review. Erratum in: J Bone Joint Surg Am. 2013 Nov 6;95(21):1987. J Bone Joint Surg Am. 2013 Dec
18;95(24):e196. PubMed PMID: 24196469.
Rubash HE, Sinha RK, Paprosky W, Engh CA, Maloney WJ. A new classification system for the
management of acetabular osteolysis after total hip arthroplasty. Instr Course Lect. 1999;48:37-42. Review. PubMed PMID: 10098026.
Question 108
Two years after undergoing right total hip arthroplasty with a large-head
metal-on-metal bearing, a 57-year-old asymptomatic woman returns for
follow-up. Radiographs reveal appropriate component position with no
osteolysis. Her serum cobalt level is 12 ppb (reference range [rr], 4.0-10.0
ug/L) and her chromium level is 11 ppb (rr, 0.7-28.0 ug/L). What is the
next step in evaluation?
1. Revision
2. MR image with metal subtraction
3. CT scan
4. Follow-up in 3 to 6 months
5. No further follow up
Question 108
Two years after undergoing right total hip arthroplasty with a large-head
metal-on-metal bearing, a 57-year-old asymptomatic woman returns for
follow-up. Radiographs reveal appropriate component position with no
osteolysis. Her serum cobalt level is 12 ppb (reference range [rr], 4.0-10.0
ug/L) and her chromium level is 11 ppb (rr, 0.7-28.0 ug/L). What is the
next step in evaluation?
1. Revision
2. MR image with metal subtraction
3. CT scan
4. Follow-up in 3 to 6 months
5. No further follow up
Metal-on-Metal Total Hip
•Any MoM THA with elevated Arthroplasty
metal ion levels (even if
asymptomatic) must undergo
further imaging (MRI vs U/S) to
assess for pseudotumor and
abductor integrity
•Patients with asymptomatic THAs
and no systemic ion toxicity may
still be undergo early revision if
progressive abductor destruction is
seen

RECOMMENDED READINGS
•Chang EY, McAnally JL, Van Horne JR, Statum S, Wolfson T, Gamst A, Chung CB. Metal-on-metal total hip arthroplasty: do symptoms
correlate with MR imaging findings? Radiology. 2012 Dec;265(3):848-57. PubMed PMID: 23047842.
•Hayter CL, Gold SL, Koff MF, Perino G, Nawabi DH, Miller TT, Potter HG. MRI findings in painful metal-on-metal hip arthroplasty. AJR Am J
Roentgenol. 2012 Oct;199(4):884-93. PubMed PMID: 22997383.
•Lombardi AV Jr, Barrack RL, Berend KR, Cuckler JM, Jacobs JJ, Mont MA, Schmalzried TP. The Hip Society: algorithmic approach to diagnosis
and management of metal-on-metal arthroplasty. J Bone Joint Surg Br. 2012 Nov;94(11 Suppl A):14-8. PubMed PMID: 23118373.
Question #121
Answer #121

This question is basically asking you for the most likely causes of LATE
instability and in this specific question stem 6 events over the past two years.
Immediately you should think infection or hardware issues such as PE wear (as
in this case). Notice the gross asymmetry of the femoral component in the
cup…Thus PE wear is your answer.
Should be center-center on all views
Question 136
•A 68-year-old patient is seen 10 years after undergoing total hip
arthroplasty; extensive wear of the polyethylene and osteolysis has
occurred (Figure 136). The polyethylene used for this procedure has
had an otherwise excellent track record. The femoral head is
zirconia ceramic. What is the most likely cause of accelerated wear?

•Infection
•Third body debris from broken wires
•Monoclinic phase transformation of zirconia
•Loosening of the cemented stem with resultant cement debris
•Macrophage-mediated osteoclastic resorption
Question 136
•A 68-year-old patient is seen 10 years after undergoing total hip
arthroplasty; extensive wear of the polyethylene and osteolysis has
occurred (Figure 136). The polyethylene used for this procedure has
had an otherwise excellent track record. The femoral head is
zirconia ceramic. What is the most likely cause of accelerated wear?

•Infection
•Third body debris from broken wires
•Monoclinic phase transformation of zirconia
•Loosening of the cemented stem with resultant cement debris
•Macrophage-mediated osteoclastic resorption
Question 136
•I got this question wrong originally. The question asks why did this ceramic head
undergo accelerated wear. There is a significant amount of osteolysis, but the
stem is not loose. Infection is unlikely 10 years postoperatively. Broken wires can
cause third body wear. However, the use of zirconia ceramics may reduce the rate
of failure in total hip arthroplasty (THA) since they have a higher mechanical
strength than alumina ceramics. The crystal structure, however, is unstable and
low-temperature ageing can occur in vitro because of phase transformation. This
increase in surface roughness can cause accelerated wear

•Haraguchi K, Sugano N, Nishii T, Miki H, Oka K, Yoshikawa H. Phase


transformation of a zirconia ceramic head after total hip arthroplasty. J Bone Joint
Surg Br. 2001 Sep;83(7):996-1000. PubMed PMID: 11603539.
Cales B. Zirconia as a sliding material: histologic, laboratory, and clinical data. Clin
Orthop Relat Res. 2000 Oct;(379):94-112. Review. PubMed PMID: 11039797.
Hernigou P, Bahrami T. Zirconia and alumina ceramics in comparison with stainless-
steel heads. Polyethylene wear after a minimum ten-year follow-up. J Bone Joint
Surg Br. 2003 May;85(4):504-9. PubMed PMID: 12793553.
Question 150
During primary total knee arthroplasty, an intraoperative fracture is an
uncommon complication.
What is the most common fracture location?
1. Patella
2. Lateral tibial plateau
3. Medial tibial plateau
4. Lateral femoral condyle
5. Medial femoral condyle
Question 150
During primary total knee arthroplasty, an intraoperative fracture is an
uncommon complication.
What is the most common fracture location?
1. Patella
2. Lateral tibial plateau
3. Medial tibial plateau
4. Lateral femoral condyle
5. Medial femoral condyle
Intraoperative Fracture in TKA
•Fractures of the femoral condyles
(medial > lateral) are the most common
type of intraoperative fracture in TKA

RECOMMENDED READINGS
•Alden KJ, Duncan WH, Trousdale RT, Pagnano MW, Haidukewych GJ. Intraoperative fracture during primary total knee arthroplasty. Clin Orthop Relat Res. 2010
Jan;468(1):90-5. PubMed PMID: 19430855.
•Sharkey PF, Hozack WJ, Booth RE Jr, Rothman RH. Intraoperative femoral fractures in cementless total hip arthroplasty. Orthop Rev. 1992 Mar;21(3):337-42. PubMed PMID:
1565523.
Question 154
Figures 154a through 154g are the radiographs and MR images of a 48-
year-old healthy man who works in construction and has left knee pain.
He is unable to climb stairs and has locking and buckling of his knee that is
worse with twisting activities. Steroid injections, anti-inflammatory drugs,
physical therapy, and bracing have failed to provide pain relief. What is the
best treatment recommendation for this patient?

1. Tibial osteotomy
2. Mensical transplant
3. Knee arthroscopy
4. Medial unicompartmental knee arthroplasty
5. Total knee arthroplasty
Question 154
Figures 154a through 154g are the radiographs and MR images of a 48-
year-old healthy man who works in construction and has left knee pain.
He is unable to climb stairs and has locking and buckling of his knee that is
worse with twisting activities. Steroid injections, anti-inflammatory drugs,
physical therapy, and bracing have failed to provide pain relief. What is the
best treatment recommendation for this patient?

1. Tibial osteotomy
2. Mensical transplant
3. Knee arthroscopy
4. Medial unicompartmental knee arthroplasty
5. Total knee arthroplasty
Question 154
• Contraindications to UKA:
– Inflammatory arthritis
– ACL deficiency
– Fixed Varus deformity >10 degrees or valgus deformity >5 degrees
– Tricompartmental OA
– Flexion contracture >10 degrees
– Unable to flex >90 degrees
• Compared to HTO, a UKA results in:
– Faster recovery
– Higher success rate initially
– Fewer short term complications
– Easier conversion to TKA

Borus T, Thornhill T. Unicompartmental Knee Arthroplasty. J Am Acad Orthop


Surg 2008; 16:9-18.
Question 165
•A 52 y/o woman underwent a cruciate-retaining total knee
arthroplasty to address a valgus knee. Although she received
adequate physical therapy during the immediate postsurgical
period, she has only 70 degrees of knee flexion. Her C-reactive
protein and erythrocyte sedimentation levels are within
defined limits. What is the most appropriate next step?
•1. Bone scan
•2. Aspiration
•3. Manipulation under anesthesia
•4. Arthroscopic release of the posterior cruciate ligament
•5. Continuous passive motion and additional physical therapy
Question 165
•A 52 y/o woman underwent a cruciate-retaining total knee
arthroplasty to address a valgus knee. Although she received
adequate physical therapy during the immediate postsurgical
period, she has only 70 degrees of knee flexion. Her C-reactive
protein and erythrocyte sedimentation levels are within
defined limits. What is the most appropriate next step?
•1. Bone scan
•2. Aspiration
•3. Manipulation under anesthesia
•4. Arthroscopic release of the posterior cruciate ligament
•5. Continuous passive motion and additional physical therapy
Arthrofibrosis
•Clinical outcomes of manipulation under anesthesia (MUA) were compared to the control TKAs who did
not develop stiffness. Total of 1671 patients are included. At a mean follow-up of 51 months (range, 24 to
85 months), the mean gains in flexion in the MUA cohort were 33° (range, 5° to 65°), although final range-
of-motion in the MUA cohort was lower than the comparison cohort (114° versus 125°). This suggests that
MUA is an effective treatment for limited flexion following primary TKA. There were no differences in the
Knee Society objective and functional scores between the two cohorts.

•RECOMMENDED READINGS
•Issa K, Kapadia BH, Kester M, Khanuja HS, Delanois RE, Mont MA. Clinical, objective, and functional
•outcomes of manipulation under anesthesia to treat knee stiffness following total knee arthroplasty. J
•Arthroplasty. 2014 Mar;29(3):548-52. doi: 10.1016/j.arth.2013.07.046. Epub 2013 Sep 4. PubMed PMID:
•24011781.
•Maniar RN, Baviskar JV, Singhi T, Rathi SS. To use or not to use continuous passive motion posttotal
•knee arthroplasty presenting functional assessment results in early recovery. J Arthroplasty. 2012
•Feb;27(2):193-200.e1. doi: 10.1016/j.arth.2011.04.009. Epub 2011 Jul 12. PubMed PMID: 21752575.
•Bedair H, Ting N, Jacovides C, Saxena A, Moric M, Parvizi J, Della Valle CJ. The Mark Coventry Award:
•diagnosis of early postoperative TKA infection using synovial fluid analysis. Clin Orthop Relat Res. 2011
•Jan;469(1):34-40. doi: 10.1007/s11999-010-1433-2. PubMed PMID: 20585914; PubMed Central PMCID:
•PMC3008895.
Question 177
What is the mechanism of action of tranexamic acid in
decreasing blood loss during joint arthroplasty
surgery?

1. Activates factor V
2. Activates factor XIII
3. Inhibits fibrinogen
4. Inhibits plasminogen
5. Blocks conversion of factor X to Xa
Question 177
What is the mechanism of action of tranexamic acid in
decreasing blood loss during joint arthroplasty
surgery?

1. Activates factor V
2. Activates factor XIII
3. Inhibits fibrinogen
4. Inhibits plasminogen
5. Blocks conversion of factor X to Xa
Tranexamic acid (TXA)
•TXA (Lysteda) is an antifibrinolytic that
promotes and stabilizes clot formation. It
competitively inhibits the activation of
plasminogen by binding to the lysine binding
site. It is associated with decreased blood
loss, drain output and transfusion
requirements in joint replacement surgery
Question 177
Recommended Readings
• Watts CD, Pagnano MW. Minimising blood loss and transfusion in contemporary
hip and knee arthroplasty. J Bone Joint Surg Br. 2012 Nov;94(11 Suppl A):8-10. doi:
10.1302/0301-620X.94B11.30618. Review. PubMed PMID: 23118371.

• Gillette BP, DeSimone LJ, Trousdale RT, Pagnano MW, Sierra RJ. Low risk of
thromboembolic complications with tranexamic acid after primary total hip and
knee arthroplasty. Clin Orthop Relat Res. 2013 Jan;471(1):150-4. doi:
10.1007/s11999-012-2488-z. PubMed PMID: 22814857; PubMed Central PMCID:
PMC3528901.

• Imai N, Dohmae Y, Suda K, Miyasaka D, Ito T, Endo N. Tranexamic acid for reduction
of blood loss during total hip arthroplasty. J Arthroplasty. 2012 Dec;27(10):1838-
43. doi: 10.1016/j.arth.2012.04.024. Epub 2012 Jun 14. PubMed PMID: 22704229.
Question 184
While performing total hip arthroplasty, a surgeon places a retractor under the
transverse acetabular ligament and encounters some brisk bleeding. Which artery
most likely has been injured?

1. Obturator
2. External iliac
3. Profunda femoris
4. Lateral femoral circumflex
5. Medial femoral circumflex
Question 184
While performing total hip arthroplasty, a surgeon places a retractor under the
transverse acetabular ligament and encounters some brisk bleeding. Which artery
most likely has been injured?

1. Obturator
2. External iliac
3. Profunda femoris
4. Lateral femoral circumflex
5. Medial femoral circumflex
Vascular dangers about the hip
Overall arterial structure – Common illiac divides at L5-S1. the anterior division becomes the external iliac and femoral artery distally. The posterior division becomes the internal iliac artery which gives off the
superior gluteal and obturator A. which then again divides into the inferior gluteal and internal pudendal arteries. The external and interal iliac A./V. are more immobile than others and lie close to the pelvis
increasing their risk of injury.

Concerning maneuvers

External iliac vein: xs reaming of the acetabulum, screw placement in anterior quadrents, careless cement placement in anterior inferior quadrant

Femoral vessles: poor retractor placement anteriorly (placement should be directly on bone), careless capsular dissection

Internal pudendal, and gluteal arteries (superior and inferior): at risk with tranacetabular screw placement (posterior inferior) or pin retractor placement near the sciatic notch.

Obturator: Careless cement placement in the anterior inferior quadrant, placement of a retractor in the obturator foramen. In the images below you can see the anatomic relationship of the transverse
acetabular ligament just superior to the obturator foramen, careless placement of a retractor in this location is most likely to injury the obturator artery of the vessels named in the possible answers.

Most common types of injury to vessels was thromboembolic followed by lacerations and then psudo-anyurysm. These are reported to occur on the left side in greater frequency than the right.

If a vsacular injury occurs and you can not get control of the bleeding a retroperitoneal approach to the acetabulum may be indicated.

RECOMMENDED READINGS

Della Valle CJ, DiCesare PE. Bulletin of the NYU Hospital for Joint Disease;June, 2002. http://www.

highbeam.com/publications/bulletin-of-the-nyu-hospital-for-joint-diseases-p136781/june-2002 Last

accessed 9/8/14

Nachbur B, Meyer RP, Verkkala K, Zürcher R. The mechanisms of severe arterial injury in surgery of the

hip joint. Clin Orthop Relat Res. 1979 Jun;(141):122-33. PubMed PMID: 477093.

Rue JP, Inoue N, Mont MA. Current overview of neurovascular structures in hip arthroplasty: anatomy,

preoperative evaluation, approaches, and operative techniques to avoid complications. Orthopedics. 2004

Jan;27(1):73-81; quiz 82-3. Review. PubMed PMID: 14763537.

Images on the next page from Rue JP et al. article


Rue JP, et al.
Question 191
• Figure 191 is the fracture radiograph of a 74-year-old woman who
underwent uncomplicated primary left total hip arthroplasty 2 weeks ago.
Shortly after surgery, she fell and sustained a periprosthetic femur
fracture. What is the most appropriate next step?
• 1. Open reduction and internal fixation using plates and screws
• 2. Open reduction and internal fixation using plates, screws, and cables
• 3. Revision of the femoral component using a cemented femoral
component
• 4. Revision of the femoral component using a distally fixing femoral
component design
• 5. Revision of the femoral component using a larger femoral component of
the same design
Figures for Question 191
Question 191
• Figure 191 is the fracture radiograph of a 74-year-old woman who
underwent uncomplicated primary left total hip arthroplasty 2 weeks ago.
Shortly after surgery, she fell and sustained a periprosthetic femur
fracture. What is the most appropriate next step?
• 1. Open reduction and internal fixation using plates and screws
• 2. Open reduction and internal fixation using plates, screws, and cables
• 3. Revision of the femoral component using a cemented femoral
component
• 4. Revision of the femoral component using a distally fixing femoral
component design
• 5. Revision of the femoral component using a larger femoral component of
the same design
Explanation for Question 191
• Periprosthetic fractures around the femoral stem is classified according to
the Vancouver Classification System
–Type A: Fracture in the trochanteric region
–Type B1: Fracture around stem, with a well fixed stem
–Type B2: Fracture around a stem that is loose but with adequate bone stock
–B3: Fracture around a stem that is loose with poor quality bone stock
–C: Fracture that occurs well below the stem
• Given that the stem was placed only 2 weeks ago, it has not had time to
become well fixed and is therefore loose and a B2 fracture
• B2 fractures are treated with revision of the femoral component to a long
porous-coated cementless stem that is distally fitting
• Must bypass the most distal cortical deficiency by two corticies
Question 206 - 2014
•Question 206 Figures 206a through 206d are the radiographs and clinical
photograph of a 90-year-old man who had knee arthroplasty 20 years ago.
He cannot fully straighten his knee. His C-reactive protein level is 1 mg/L
(reference range [rr], 0.08-3.1 mg/L) and his erythrocyte sedimentation
rate is 10 mm/h (rr, 0-20 mm/h). Knee aspiration reveals 500 WBC/mm3
with 40% neutrophils. What is the most likely cause of his condition?

•1. Infection
•2. Instability
•3. Implant loosening
•4. Malalignment
•5. Patella maltracking
Question 206 Image
Question 206 – 2014 Hip and Knee
•Question 206 Figures 206a through 206d are the radiographs and clinical
photograph of a 90-year-old man who had knee arthroplasty 20 years ago.
He cannot fully straighten his knee. His C-reactive protein level is 1 mg/L
(reference range [rr], 0.08-3.1 mg/L) and his erythrocyte sedimentation
rate is 10 mm/h (rr, 0-20 mm/h). Knee aspiration reveals 500 WBC/mm3
with 40% neutrophils. What is the most likely cause of his condition?

•1. Infection
•2. Instability
•3. Implant loosening
•4. Malalignment
•5. Patella maltracking
Question 206 Explanation
Controversy remains whether or not patellar resurfacing should be performed
during TKA. A rare complication is aseptic patellar loosening and extra-
articular migration of the patella button. This is eventually treated with patella
component removal without replacement.

A meta-analysis comparing resurfaced and non-resurfaced patella


demonstrated similar pain and functional outcomes, but the rate of
complications and re-operation was significantly higher in the non-resurfaced
group.

[Whey do TKA fail? 1. aseptic loosening. 2. instability. 3. infection.]

Jacobs E, Feczko P, Emans P. J Knee Surg. 2013 Dec;26 Suppl 1:S100-2. doi:10.1055/s-0032-1322601. Epub
2012 Jul 30. PubMed PMID: 23288755.
Pilling RW, Moulder E, Allgar V, et al. J Bone Joint SurgAm. 2012 Dec 19;94(24):2270-8. doi: 10.2106/
JBJS.K.01257. PubMed PMID: 23318618.
Schroer WC, Berend KR, Lombardi AV, et al. J Arthroplasty. 2013 Sep;28(8 Suppl):116-9. doi:
10.1016/j.arth.2013.04.056. Epub 2013 Aug 15. PubMed PMID: 23954423.
Question 236
Question 236: Answer
Question 236: Explanation
• AAOS clinical practice guidelines based on Level I
and II evidence

• Rationale: 2 Level I studies compares


vertebroplasty to sham: no difference in pain or
function at 1 and 6 months

• Level II studies revealed improvement in pain at


24h but no significant improvement after 6 weeks.
Question 248
Question 248

Figures 248a and 248b are the radiographs of an 80-year-old woman who had total knee arthroplasty 15
years ago. The procedure had been working well until 1 week ago when she heard a pop while standing
from a chair. An initial examination reveals she cannot extend her knee. After aspirating 50 cc of bloody
fluid and injecting the knee with lidocaine, she can extend the knee against gravity but not against
resistance. There is no palpable gap in her quadriceps or patella tendon. What is the most appropriate
treatment?

1. Patellectomy
2. Extensor mechanism allograft
3. Primary repair of the tendon rupture
4. Nonsurgical treatment with a knee immobilizer
5. Physical therapy to improve quadriceps strength
Question 248 IMAGE
Question 248 ANSWER
Question 248

Figures 248a and 248b are the radiographs of an 80-year-old woman who had total knee arthroplasty 15
years ago. The procedure had been working well until 1 week ago when she heard a pop while standing
from a chair. An initial examination reveals she cannot extend her knee. After aspirating 50 cc of bloody
fluid and injecting the knee with lidocaine, she can extend the knee against gravity but not
against resistance. There is no palpable gap in her quadriceps or patella tendon. What is the most
appropriate treatment?

1. Patellectomy
2. Extensor mechanism allograft
3. Primary repair of the tendon rupture

4. Nonsurgical treatment with a knee immobilizer


5. Physical therapy to improve quadriceps strength
Question 248 EXPLANATION
• Prevalence, complications, and outcomes. JBJS 2005 PMID15634812.
– The prevalence of a quadriceps tendon tear after total knee arthroplasty was 0.1% (twenty-four of 23,800).
– CONCLUSION: The prevalence of complications was high, and the outcomes were poor for seven of the eleven
patients who had a complete quadriceps tendon tear after total knee arthroplasty. Patients who sustained a
partial tear and were treated nonoperatively had no complications and had uniformly good
outcomes.

• Management of extensor mechanism rupture after TKA. JBJS Br 2012 PMID: 23118397
– Disruption of the extensor mechanism in total knee arthroplasty may occur by tubercle avulsion, patellar or quadriceps tendon rupture, or
patella fracture, and whether occurring intra-operatively or post-operatively can be difficult to manage and is associated with a significant rate
of failure and associated complications. This surgery is frequently performed in compromised tissues, and repairs must frequently be
protected with cerclage wiring and/or augmentation with local tendon (semi-tendinosis, gracilis) which may also be used to treat soft-tissue
loss in the face of chronic disruption.
– Quadriceps rupture may be treated with conservative therapy if the patient retains active
extension.
– Component loosening or loss of active extension of 20° or greater are clear indications for surgical treatment of patellar fracture. Acute
patellar tendon disruption may be treated by primary repair. Chronic extensor failure is often complicated by tissue loss and retraction can be
treated with medial gastrocnemius flaps, achilles tendon allografts, and complete extensor mechanism allografts. Attention to fixing the graft
in full extension is mandatory to prevent severe extensor lag as the graft stretches out over time.
Question 248
Question 248

Figures 248a and 248b are the radiographs of an 80-year-old woman who had total knee arthroplasty 15
years ago. The procedure had been working well until 1 week ago when she heard a pop while standing
from a chair. An initial examination reveals she cannot extend her knee. After aspirating 50 cc of bloody
fluid and injecting the knee with lidocaine, she can extend the knee against gravity but not against
resistance. There is no palpable gap in her quadriceps or patella tendon. What is the most appropriate
treatment?

1. Patellectomy
2. Extensor mechanism allograft
3. Primary repair of the tendon rupture
4. Nonsurgical treatment with a knee immobilizer
5. Physical therapy to improve quadriceps strength
Question 248 IMAGE
Question 248 ANSWER
Question 248

Figures 248a and 248b are the radiographs of an 80-year-old woman who had total knee arthroplasty 15
years ago. The procedure had been working well until 1 week ago when she heard a pop while standing
from a chair. An initial examination reveals she cannot extend her knee. After aspirating 50 cc of bloody
fluid and injecting the knee with lidocaine, she can extend the knee against gravity but not
against resistance. There is no palpable gap in her quadriceps or patella tendon. What is the most
appropriate treatment?

1. Patellectomy
2. Extensor mechanism allograft
3. Primary repair of the tendon rupture

4. Nonsurgical treatment with a knee immobilizer


5. Physical therapy to improve quadriceps strength
Question 248 EXPLANATION
• Prevalence, complications, and outcomes. JBJS 2005 PMID15634812.
– The prevalence of a quadriceps tendon tear after total knee arthroplasty was 0.1% (twenty-four of 23,800).
– CONCLUSION: The prevalence of complications was high, and the outcomes were poor for seven of the eleven
patients who had a complete quadriceps tendon tear after total knee arthroplasty. Patients who sustained a
partial tear and were treated nonoperatively had no complications and had uniformly good
outcomes.

• Management of extensor mechanism rupture after TKA. JBJS Br 2012 PMID: 23118397
– Disruption of the extensor mechanism in total knee arthroplasty may occur by tubercle avulsion, patellar or quadriceps tendon rupture, or
patella fracture, and whether occurring intra-operatively or post-operatively can be difficult to manage and is associated with a significant rate
of failure and associated complications. This surgery is frequently performed in compromised tissues, and repairs must frequently be
protected with cerclage wiring and/or augmentation with local tendon (semi-tendinosis, gracilis) which may also be used to treat soft-tissue
loss in the face of chronic disruption.
– Quadriceps rupture may be treated with conservative therapy if the patient retains active
extension.
– Component loosening or loss of active extension of 20° or greater are clear indications for surgical treatment of patellar fracture. Acute
patellar tendon disruption may be treated by primary repair. Chronic extensor failure is often complicated by tissue loss and retraction can be
treated with medial gastrocnemius flaps, achilles tendon allografts, and complete extensor mechanism allografts. Attention to fixing the graft
in full extension is mandatory to prevent severe extensor lag as the graft stretches out over time.
Question 258
Which bearing couple is associated with the least volumetric wear?

1. Metal on metal

2. Metal on highly cross-linked polyethylene

3. Metal on ceramic

4. Ceramic on ceramic

5. Ceramic on highly cross-linked polyethylene


Question 258
Explanation
4. Ceramic on ceramic
–Ceramic couplings have lower wear
rates due to their lower coefficient of
friction compared to the other
options.
–Ceramics also have low grain size,
high hardness leading to scratch
resistance, high wettability, and fluid-
film lubrication.
Suggested Reading

•Hannouche D, Manadouche M, Nizard R, Bizot P, Meunier A, Sedel L.


Ceramics in Total Hip Replacement. Clin Orthop Relat Res. 2005; 430:62-71

•Garino JP. Modern ceramic-on-ceramic total hip systems in the United


States: early results. Clin Orthop Relat Res. 2000; 379:41-7. PubMed PMID:
11039791.

•Clarke IC, Donaldson T, Jobe C. Impact of wear debris on success of total hip
replacements. In: Garino JP, Beredjiklian PK, eds. Core Knowledge in
Orthopaedics: Adult Reconstruction and Arthroplasty. Mosby/Elsevier;2007.
Question 264
What is the best predictor of pain for patients with hip
osteonecrosis?
1. Ficat stage II disease
2. Bone marrow edema
3. Bilateral hip involvement
4. Modified Kerboul angle less than 190 degrees
5. Use of oral bisphosphonates
Question 264
What is the best predictor of pain for patients with hip
osteonecrosis?
1. Ficat stage II disease
2. Bone marrow edema
3. Bilateral hip involvement
4. Modified Kerboul angle less than 190 degrees
5. Use of oral bisphosphonates
Recommended Reading
Paper below shows that bone marrow edema on MRI is highly correlated with symptomatic AVN of the hip and the strongest predictor of likelihood of
worsening pain from hip AVN.

Steinberg Classification (modification of Ficat)

Stage 0 – normal xray and normal MRI/bone scan


Stage I – normal xray, abnormal MRI/bone scan
Stage II – cystic or sclerotic changes
Stage III – crescent sign (subchondral collapse)
Stage IV – flattening (collapse) of femoral head
Stage V – narrowing of joint
Stage VI – advanced degenerative changes

Modified Kerboul angle - Draw two radii from the center of the femoral head to encompass the lesion.
Combine the angle these radii create on the AP and Lateral view to obtain angle

100% of hips > 240°collapsed, 50% of hips between 190°and 240°collapsed


and 0% of hips < 190°collapsed (paper below)

Bisphosphonates – some trials have shown that progression from stages 0-II to femoral head collapse
can be prevented by alendronate but other studies have shown no benefit

•Ito H, Matsuno T, Minami A. Relationship between bone marrow edema and development of symptoms in patients with osteonecrosis of the femoral head. AJR Am J Roentgenol. 2006 Jun;186(6):1761-70. PubMed PMID: 16714671.
•Ha YC, Jung WH, Kim JR, Seong NH, Kim SY, Koo KH. Prediction of collapse in femoral head osteonecrosis: a modified Kerboul method with use of magnetic resonance images. J Bone Joint Surg Am. 2006 Nov;88 Suppl 3:35-40. PubMed
PMID: 17079365.

You might also like